Path GI tract Flashcards Preview

CDHB Path > Path GI tract > Flashcards

Flashcards in Path GI tract Deck (197)
Loading flashcards...
1
Q

1.5cm long distal oesophageal mass is biopsied yielding diagnosis of adenocarcinoma. Origin of lesion most likely ? Rob p553

1.Barrett’s oesophagus
2.Extension from adjacent Ca lung
3.Extension from fundal Ca stomach

A

1.Barrett’s oesophagus

1.Barrett’s oesophagus Adenocarcinoma : 30 - 40X increased risk with > 2cm of Barrett mucosa. UTD: most if not all oesophageal adenocarcinoma arises from Barrett’s metaplasia.

2
Q

2.Tracheo-oesophageal fistula, MOST COMMON ? Rob p549

1.Proximal fistula, distal blind pouch
2.“H” type
3.Distal fistula, proximal pouch
4.Misnomer because no atresia
5.Misnomer because no fistula

A

3.Distal fistula, proximal pouch

2.Tracheo-oesophageal fistula, MOST COMMON ? Rob p (TW)

1.Proximal fistula, distal blind pouch
2.“H” type
3.Distal fistula, proximal pouch - T - oesophageal atresia and tracheo oesophageal fistula not in communication with proximal oesophagus (82%)
4.Misnomer because no atresia
5.Misnomer because no fistula

3
Q

3.Classic appearance of Malloy Weiss tear is ? Rob p550

1.Linear at gastro-oesphageal junction
2.Linear with hiatus hernia
3.Undermined proximal mucosa
4.Circular at GOJ
5.Circular with

A

1.Linear at gastro-oesphageal junction

3.Classic appearance of Malloy Weiss tear is ? Rob p768 (–)

1.Linear at gastro-oesphageal junction (linear lacerations in axis of oesophagus lumen, at or below the G-E junction)
2.Linear with hiatus hernia
3.Undermined proximal mucosa
4.Circular at GOJ
5.Circular with
Note : Boerhaave syndrome = oesophageal rupture (rare)

4
Q

4.What are risk factors for malignancy in a stomach ulcer ?

1.pernicious anaemia
2.Crohn’s disease
3.coeliac disease
4.H. Pylori

A

Answer: H.pylori. Probably old questions (see below)

ROBBINS
Malignant transformation of peptic ulcers occurs rarely, if ever, and reports of transformation probably represetn cases in which a lesion thought to be a chronic peptic ulcer was actually an ulcerated carcinoma from the start.

STATDX
2 major risk factors for gastric ulcer (malignant or benign): Helicobacter pylori, NSAID use.

5
Q

5.Malignant gastric ulcer. What would be the indicator of malignancy? Rob p561
1.> 4cm diameter
2.single ulcer of greater curvature
3.heaped edges

A

3.heaped edges

5.Malignant gastric ulcer. What would be the most concerning for malignancy? Rob p780 (TW)

1.> 4cm diameter - size does not diffentiate a benign from malignant ulcer, usually less than 2cm)
2.single ulcer of greater curvature – Robbins p780 – size & location do not differentiate benign & malignant ulcers. location of gastric carcinoma within stomach is: pylorus and antrum 50-60%; cardia 25%; remainder in body an fundus. The lesser curvature is involved in about 40% and the greater curvature in 12%. Thus, a favored location is the lesser curvature of the antropyloric region. Although less frequent, an ulcerative lesion on the greater curvature is more likely to be malignant.
3.heaped edges - heaping up of the margins is rare in the benign ulcer but is characteristic of the malignant lesion.

6
Q

6.Stomach cancer, which is NOT TRUE? Rob p561

1.Benign ulcers on lesser curvature at antrum
2.Alcohol is not a risk factor
3.Most common is adeno-carcinoma/ then lymphoma/ then carcinoid/then GIST
4.Early gastric cancer is confined to the mucosa/submucosa with negative perigastric nodes
5.Macro – excavated, exophytic,flat

A

4.Early gastric cancer is confined to the mucosa/submucosa with negative perigastric nodes

6.Stomach cancer, which is NOT TRUE? Rob p784 (TW)

1.Benign ulcers on lesser curvature at antrum - T - gastric ulcers are predominantly located along the lesser curvature, in or around the border zone between the corpus and the antral mucosa.

2.Alcohol is not a risk factor - T

3.Most common is adenocarcinoma/ then lymphoma/ then carcinoid/then GIST - T - Adeono 90-95%, Lymphoma 4%, Carcinoids 3%, malignant stromal cell tumours (2%).

4.Early gastric cancer is confined to the mucosa/submucosa with negative perigastric nodes - F - regardless (ie not influenced by) of presence of perigastric nodes (Robbins).

5.Morphological types – excavated, exophytic or flat (true)

Note : Ulcerated carcinomas usually have nodular raised margins with “dirty” necrotic bases, and lack surrounding radiating folds
Early gastric carcinoma: lesion confined to mucosa and submucosa, regardless of presence or absence of perigastric lymph node metastases.
Advanced gastric carcinoma: neoplasm that has extendd below the submucos into the muscular wall.

**LJS - Depth of invasion and extent of nodal spread/mets at dx most imp indicator of prognosis
If found early and surgical resection still possible, 5YS 90% even if nodal mets present

7
Q

7.Which of the following statements concerning GIT diseases IS INCORRECT:

1.Most gut lymphomas are T-cell type
2.Villous adenomas are most common in the rectum
3.In Hirshsprung’s Disease, the rectum is always involved
4.Angiodysplasia is most common in the proximal colon
5.Necrotising enterocolitis is most common in neonates

A

1.Most gut lymphomas are T-cell type

7.Which of the following statements concerning GIT diseases IS INCORRECT: (JS)

1.Most gut lymphomas are T-cell type - F - most are B cell lymphomas (90%)
2.Villous adenomas are most common in the rectum - T - only 1% of adenomatous polyps, more common in rectum and rectosigmoid but can occur anywhere.
3.In Hirshsprung’s Disease, the rectum is always involved - T
4.Angiodysplasia is most common in the proximal colon - T - most common in caecum and right colon
5.Necrotising enterocolitis is most common in neonates - T

8
Q

8.Coeliac disease. What is the least likely?

1.anti-gliadin antibodies
2.treatment prevents development of lymphoma

A

2.treatment prevents development of lymphoma

8.Coeliac diseasease What is the least likely? (JS)

1.anti-gliadin antibodies - T - due to a sensitivity to gluten (gliadin)

2.treatment prevents development of lymphoma - F - there is a long term risk of malignant disease at 2 times the usual rate - intestinal lymphomas (including disproportionately high number of T-cell lymphomas) and GIT and breast cancers.
SK – UTD 2011 – “Whether the degree of compliance with a gluten-free diet influences the rates of these cancers is uncertain. In one study, the increased risk of non-Hodgkin lymphoma persisted for five years after diagnosis despite adherence to a gluten-free diet”

9
Q

9.Celiac disease associated with?
1.Less than 2x risk of malignancy
2.Bowel wall thinning

A

*LW:
- Option stated LESS than 2x risk of cancer, most of the large research studies quote risks > 2, so this answer is favored to be FALSE.
- Bowel wall thinning, seems a non specific term, but given villi are part of the bowel wall, and there is villous atrophy, I would favour this to be correct.

1.Less than 2x risk of malignancy

9.Coeliac disease associated with? Rob p571 (JS)

1.2x risk of malignancy - T - there is a long term risk of malignant disease at 2 times the usual rate - intestinal lymphomas (including disproportionately high number of T-cell lymphomas) and GIT and breast cancers.
SK – studies give variable numbers for increased risk of malignancy, would be safe with at least double risk (more like 5 x’s for lymphoma)

2.Bowel wall thinning - F - crypts are elongated, hypoplastic and tortuous but the overall mucosal thickness remains the same

**LJS - Rob just says incr lymphoma/adenoca, no indictation of how much.
Loss of mucosal and brush border surface area = malabsorption

10
Q

10.Coeliac disease, most correct statement is
1.definitive diagnosis is shown by showing histological total villous atrophy
2.10-20% have anti-gliadin Ab’s without having the disease

A

Answer: Unsure, probable old question, incomplete recall. See below

FROM ROBBINS
Histopathology characterized by increased numbers of intraepithelial CD8+ T lymphoctyes, crypt hyperplasia, and villous atrophy.

With increased serologic screening and early detection of disease-associated antibodies, it is now appreciated that increased numbers of intraepithelial lymphocytes, particularly within the villus, is a sensitive marker of celiac disease, even in the absence of epithelial damage and villous atrophy. However, intraepithelial lymphocytosis and villous atrophy are not specific for celiac disease and can be present in other diseases, including viral enteritis.

With increased serologic screening and early detection of disease-associated antibodies, it is now appreciated that increased numbers of intraepithelial lymphocytes, particularly within the villus, is a sensitive marker of celiac disease, even in the absence of epithelial damage and villous atrophy. However, intraepithelial lymphocytosis and villous atrophy are not specific for celiac disease and can be present in other diseases, including viral enteritis.

PATH OUTLINES
Gold standard for diagnosis is serology with confirmation of histology on duodenal biopsy
TGA has the highest sensitivity for celiac disease (98%); specificity is 90%.
Positive even in individuals on gluten free diet

11
Q

11.Which of the following statements concerning gastrointestinal diseases is most correct?
1.The colon is the most common site for GIT lymphoma
2.Pseudomembranous colitis is due to toxins of clostridium difficile
3.Inflammatory pseudo-polyps are a characteristic feature of Crohn’s colitis
4.Apthous ulcers are a characteristic feature of ulcerative colitis
5.Juvenile polyps typically occur in the ascending colon.

A

2.Pseudomembranous colitis is due to toxins of clostridium difficile

11.Which of the following statements concerning gastrointestinal diseases is most correct? (JS)
1.The colon is the most common site for GIT lymphoma - F - Stomach (55-60%), small intestine (25-30%), proximal colon (10-15%) and distal colon (10%) with appendix and oesophagus rarely involved.
2.Pseudomembranous colitis is due to toxins of clostridium difficile - T - Caused by exotoxins A and B of C. difficle after antibiotic therapy
3.Inflammatory pseudo-polyps are a characteristic feature of Crohn’s colitis - F - associated with UC but can occur in Crohns
4.Apthous ulcers are a characteristic feature of ulcerative colitis - F - associated with Crohns
5.Juvenile polyps typically occur in the ascending colon. - F - hamartomatous malformations of mucosa, typically in the rectum

12
Q

12.In which one of the following are granulomas NOT a characteristic feature?
1.Crohn’s disease
2.Sarcoidosis
3.Foreign body response
4.Ulcerative colitis
5.Tuberculosis

A

4.Ulcerative colitis

12.In which one of the following are granulomas NOT a characteristic feature? (JS/SK)
1.Crohn’s disease – found in 50-60% (characteristic??), non-caseating
2.Sarcoidosis T non-caseating
3.Foreign body response T
4.Ulcerative colitis F
5.Tuberculosis T caseating

13
Q

13.Which of the following IS NOT a recognised feature of diverticular disease of the colon:
1.Hypertrophy of the circular muscle layer of the bowel
2.Dissection into the appendices epiploicae
3.Crypt abscesses
4.Fistulae
5.Haemorrhage

A

3.Crypt abscesses

13.Which of the following IS NOT a recognised feature of diverticular disease of the colon: (JS)
1.Hypertrophy of the circular muscle layer of the bowel - T
2.Dissection into the appendices epiploicae - T
3.Crypt abscesses - F - Typical lesion of Crohn - collections of neutrophils within crypt lumens
4.Fistulae - T
5.Haemorrhage - T

14
Q

14.Concerning carcinoid tumours of the gastro-intestinal tract, which of the following statements IS LEAST correct:
1.The appendix is the most common site
2.They have a characteristic desmopiostic reaction
3.They are benign tumours
4.A characteristic feature is a solid, yellow-tan appearance on transections
5.They are a more common small bowel neoplasm than primary adenocarcinomas

A

Answer: The appendix is the most common site < they are benign tumors.

Probable incorrect recall given 2 wrong answers.

ROBBINS: GI NEUROENDOCRINE
- intramural or submucosal masses that create polypoid lesions
- Yellow or tan in colour, and very firm as a consequence of intense desmoplasia
- can be subdivided into neuroendocrin tumours and neuroendocrine carcinomas on the basis of histology and immunohistochemistry for Ki-67
- Small intestine tumours tend to be most aggressive, while those of the appendix are typically benign.

15
Q

15.Amebiasis which is TRUE ? Rob p314,569 Dan p791
1.Organsism is Ngareri foweri
2.Invasion through mucosa into Peyers patches
3.10% of infected people have dysentry
4.cysts release trophozoites secondary to pancreatic enzymes (? Pancreatic ?)
5.release exotoxin and denude epithelium

A
  1. 10% of infected people have dysentry

15.Amebiasis which is TRUE ? Rob p806 (JS)
1.Organism is Ngareri Fowleri - F - Entamoeba histolytica
2.Invasion through mucosa and into peyers patches - F - invade the crypts of the colonic glands and burrow through the lamina propria and are halted by the muscularis mucosae.
3.10% of infected people have dysentery - T
4.Cysts release trophozoites secondary to pancreatic enzymes. - F - release trophozoites under anaerobic conditions in colon (cysts resistant to gastric acid)
5.Release exotoxin and denude epithelium - F - This is referring to C difficle in pseudomembranous colitis. Amoebae produce proteins involved in tissue invasion including proteinases, lectin and amebapore, resulting in ulcers and sloughing of the mucosa

16
Q

16.Crohn’s disease in counselling a sibling of risk
1.1% risk
2.No increased risk
3. 10% risk.

A
  1. 10% risk.

16.Crohn’s disease in counselling a sibling of risk (JS)
1.1% risk
2.No increased risk
3.10% risk - T - lifetime risk if either a parent or sibling is affected is 9% (Robbins): PathOutlines 15% have affected first degree relatives

17
Q

17.Crohns disease is NOT ASSOCIATED with ?
1.Hydronephrosis
2.Carcinoma
3.Hip arthopathy
4.Sclersoing cholangitis
5.Sclerosing peritonits

A

Answer: Sclerosing peritonitis

18
Q

18.Patient with ulcerative colitis with elevated CEA (Carcinoembryonic antigen)? Rob p574, 208,585
1.Likely colon cancer
2.May be colon cancer but also raised in UC active inflammation
3.Active ulcerative Colitis

A

2.May be colon cancer but also raised in UC active inflammation

18.Patient with ulcerative colitis with elevated CEA (Carcinoembryonic antigen)? Rob p574, 208,585 (JS)
1.Likely colon cancer
2.May be colon cancer but also raised in UC active inflammation - T - produced in embryonic tissue of the gut, pancreas and liver. Elevated in 60-90% of colorectal, 50-80% of pancreatic and 25-50% of gastric and breast cancer. Also elevated in cirrhosis, hepatitis, UC, Crohns and in smokers. Lacks specificity and sensitivity for early cancer detection but can be used in prognosis and detection of recurrence.
3.Active ulcerative Colitis

19
Q

19.Which does not cause bowel obstruction? Rob p578

1.Tuberous Sclerosis
2.GVHD
3.Collagen vascular disorders
4.Viral infections
5.Ingested toxins

A

3.Collagen vascular disorders (scleroderma, dermatomyositis – muscle dysfunction – marked dilatation of small bowel simulating small bowel obstruction)

19.Which does not cause bowel obstruction? mRob (–)
1.Tuberous Sclerosis ?? by leiomyoma ??
2.GVHD
3.Collagen vascular disorders (scleroderma, dermatomyositis – muscle dysfunction – marked dilatation of small bowel simulating small bowel obstruction)

20
Q

20.In colonic polyps which is TRUE ? Rob p579

1.Peutz Jeger is sporadic hamartomatous polyps
2.HNPCC (Hereditary Nonpolyposis Colorectal Cancer) not associated with adenoma
3.Most common type in adults is villous
4.Juvenille polyps occur in ileum

A

2.HNPCC (Hereditary Nonpolyposis Colorectal Cancer) not associated with adenoma
*LW: probably most true out of options listed:
Robbins states: HNPCC…adenomas occur in low numbers and considerably earlier than in genral population, however colonic malignancies that develop are multiple and not usually associated with pre exisiting adenomas. Hallmark is mutations in DNA repair genes.

20.In colonic polyps which is TRUE ? Rob p815 (JS)

1.Peutz Jeger is sporadic hamartomatous polyps - F - Autosomal dominant syndrome (not sporadic) characterised by multiple hamartomatous polyps
2.HNPCC (Hereditary Nonpolyposis Colorectal Cancer) not associated with adenoma – T/F -
JS: HNPCC is characterised by familial carcinoma of the colon, affecting predominantly caecum and proximal colon, which DON’T arise within adenomatous polyps.
SK: Robbins & Mayo both mention adenomatous polyps occurring in HNPCC. Other reference (SemSurgOnc 1995) “It appears that HNPCC patients form adenomas at about the same rate as the general population and there is circumstantial evidence that adenoma is the precursor to colorectal carcinoma in the syndrome. It is hypothesized that HNPCC features accelerated progression from colonic adenoma to carcinoma”
3.Most common type in adults is villous - F - Tubular adenomas are the most common (90%) followed by tubulovillous (5-10%) then villous (1%)
4.Juvenille polyps occur in ileum F - Focal hamartomatous polyps found most frequently in the rectum in children younger than 5y

21
Q

21.Adenomatous polyps are NOT a feature of ? Rob p820

1.Gardners Syndrome
2.Familial Polyposis
3.Turcot’s Syndrome
4.Peutz-Jeghers Syndrome
5.Sessile Villous lesions

A

4.Peutz-Jeghers Syndrome

21.Adenomatous polyps are NOT a feature of ? (JS) Rob p815

1.Gardners Syndrome - F - AD variant of FAP with intestinal polyps, osteomas and soft tissue tumours
2.Familial Polyposis - F - AD disorder with innumerable adenomatous polyps with progression to adenocarcinoma in 100%
3.Turcot’s Syndrome - F - rare variant of Gardners syndrome with colonic polyposis and tumours of the CNS
4.Peutz-Jeghers Syndrome - T - hamartomatous lesions which don’t have malignant potential themselves, but patients have an increased risk of colon & other cancers (pancreas, breast, lung, ovary and uterus)
5.Sessile Villous lesions - F - type of adenomatous polyps

22
Q

22.Following associated with increased risk of bowel malignancy EXCEPT? Rob p579

1.Obesity
2.Diabetes
3.Crohns
4.Meat/low fibre diet

A

2.Diabetes

22.Following associated with increased risk of bowel malignancy EXCEPT? Rob p579 (JS)

1.Obesity
2.Diabetes - T
3.Crohns
4.Meat/low fibre diet
Risk factors for colorectal cancer include diet (low fibre, high intake of refined carbs, intake of red meat, reduced intake of protective micronutrients such as vitamins A, C and E), obesity, physical inactivity, family history, IBD, previous XRT, FAP, HNPCC.

23
Q

23.A 60 year old male has colonic carcinoma which involves muscularis mucosa. No regional lymph nodes, no distant metastasis. What is prognosis? Rob p582

1.100% 5 year survival
2.95% 5 year survival
3.60% 5 year survival
4.40% 5 year survival
5.10% 5 year survival

A

3.60% 5 year survival

23.A 60 year old male has colonic carcinoma which involves muscularis mucosa. No regional lymph nodes, no distant metastasis. What is prognosis? Rob p582 (–) (GC & TW)

1.100% 5 year survival
2.95% 5 year survival
3.67% 5 year survival B1 = 67% (path notes); TNM staging is 90%
4.40% 5 year survival
5.10% 5 year survival

*LW:
RP Dukes staging prognosis:
Overall 5-year survival rate is 40-50%, with the stage at operation the single most important factor affecting prognosis.

Duke A: 80-90%
Duke B: 70%
Duke C: 33%
Duke D: 5%

This tumour is T1 or Duke B1

STAGING
Single most important prognostic indicator = extent of tumour at time of diagnosis

Modified Dukes Classification / Astler-Coller Staging:
A Limited to the mucosa (TNM – ‘T is’ (in situ))
B1 Extending into but not through muscularis propria, no nodal spread (TNM – T1)
B2 Through muscularis propria, no nodal spread (TNM – T2)
C1 Into muscularis propria + nodal spread (ie B1 + nodes)
C2 Through muscularis propria + nodal spread (ie B2 + nodes)
D Distant metastases
Staging can only be applied after resection of neoplasm and surgical exploration
Recurrences most common (10%):
at operation site, near anastomoses
in peritoneal cavity
in liver and distant organs
PROGNOSIS
Related to stage:
A = ~100% 5YS
B1 = 67% MR says for TNM staging prognosis is 90%
B2 = 54% MR says for TNM is 78%
C1 = 43%
C2 = 23%
Overall 5 year survival 40 – 50%

24
Q

25.Histologically appendicitis characterised by ? Rob p588

1.Neutrophils in the mucosa
2.Neutrophils in the the muscularis propria
3.Plasma cells/lymphocytes in muscularis
4.Serosal hyperaemia

A

2.Neutrophils in the the muscularis propria

25.Histologically appendicitis characterised by ? Rob p588 (JS)

1.Neutrophils in the mucosa
2.Neutrophils in the the muscularis propria - T - Histologic criterion is neutrophilic infiltration of muscularis. Usually neutrophils and ulceration is also present within the mucosa but other conditions can cause mucosal infiltrate (ie spread from GIT infection elsewhere)
3.Plasma cells/lymphocytes in muscularis
4.Serosal hyperaemia - F - is present but not specific

25
Q

26.A fifty year old female develops a colovaginal fistula. Ten years earlier she had treatment for choriocarcinoma and colonoscopy six months ago was normal. Most likely cause is ? Dan p857

1.Radiotherapy change
2.Colon Cancer
3.Crohns
4.Diverticular abscess
5.Bladder Cancer

A

4.Diverticular abscess

26.A fifty year old female develops a colovaginal fistula. Ten years earlier she had treatment for choriocarcinoma and colonoscopy six months ago was normal. Most likely cause is ? (TW)

1.Radiotherapy change - choriocarcinoma treated with surgery and chemotherapy. Dysgerminoma however can get XRT.

2.Colon Cancer - normal scope 6/12 ago.

3.Crohns - normal scope 6/12 ago. Most cases of CD and UC have onset between ages of 15 and 40yo. Many suggest a second peak between 50 and 80yo. Approx 80% pts have small bowel involvement, 50% iliocolitis, 20% limited to colon.

4.Diverticular abscess - prevalence of diverticular disease of colon: 5% at age 40yo, 30% age 60, 65% age 85yo.
*LW: only thing that makes me uneasy about this is a normal scope 6months prior, as one would expect background vierticular disease to be present before development of a diverticular abscess and resulting fistula. However, probability states this would be most likely.
*AJL: I also feel uneasy about this. Why have they mentioned cholangiocarcinoma? I wonder if it is to see if you can get the connectiong back to crohns (cholangio - PSC - IBD - Crohns (even though UC is more commonly associated). Though I agree statistically diverticular disease is more common, both crohns and diverticular disease leading to fistula would presumably have a slightly abnormal colonoscopy 6 months earlier?
**LJS - agree (although Q mentions choriocarcinoma rather than cholangio)

5.Bladder Cancer - more frequent in men than women. 80% Dx after age of 60yo.

Note : Manifestation of radiation colitis can occur up to 15 years following radiation
Common causes of fistulas iatrogenic,diverticula,Crohns; less common – amoebiasis,

26
Q

27.Risk of Caecal carcinoma :

  1. 1% synchronous
  2. 0.1% synchronous
  3. 1% synchronous & metachronous
  4. 39% in the caecum and ascending colon
  5. 10% synchronous
A

27.Risk of Caecal carcinoma : (TW)

  1. 1% synchronous - rate of synchronous CRC is about 5%.
  2. 0.1% synchronous
  3. 1% synchronous & metachronous - metachronous carcinoma occured in 2% of total, so metachronous and synchronous would be about 0.1%
  4. 39% in the caecum and ascending colon - (true for BR) *LW Robbins states 22%
  5. 10% synchronous
    Info from Japanese Journal of Clinical Oncology 2003
27
Q

28.HNPCC – barium enema is performed to look for

1.colonic carcinoma
2.polyps
3.detect colonic hamartoma

A

28.HNPCC – barium enema is performed to look for (TW)

1.Colonic carcinoma - T - HNPCC (Lynch syndrome) an AD condition, characterised by familial carcinomas of the colon, predominantly affecting the caecum and proximal colon.
2.polyps - N = nonpolyposis
3.detect colonic hamartoma - Peutz Jeghers

HNPCC / Lynch sydrome is the most common of the inherited colon susceptibility syndromes. Significantly increase risk of colon cancer and endometrial cancer as well as small risk of sever other associated cancers (ovariian, upper GUT, gastric, small bowel, biliary/pancreatic, skin, brain).
Lifetime risk of developing CRC in Lynch = 70%. Like most sporadic CRCs, Lynch cancers appear to evolve from adenomas, but when detected the adenomas tend to be larger, flatter, and are oftenmore proximal.
Accelerated adenoma-carcinoma sequence, dysplasia in flat mucosa of colon.
There are several types of DNA damage, and, correspondingly, there are many forms of DNA repair.
HNPCC results from defects in genes involved in DNA mismatch repair.
DNA repair genes themselves are not oncogenic, but they allow mutations in other genes during the process of normal cell division

28
Q

29.Patient with suspected mononucleosis, which would be atypical?

1.Abnormal LFT
2.Proteinuria with viral inclusions in epithelial cells in urine
3.Groin lymphadenopathy
4.Mononuclear meningitis
5.Atypical lymphocytes on FNA

A

2.Proteinuria with viral inclusions in epithelial cells in urine

29.Patient with suspected mononucleosis, which would be atypical? (JS)

1.Abnormal LFT - F - commonly causes hepatic dysfunction
2.Proteinuria with viral inclusions in epithelial cells in urine - T - this occurs in CMV infection
3.Groin lymphadenopathy - F - presents with fever, sore throat and lymphadenopathy
4.Mononuclear meningitis - F - can occur - not sure if it is “typical”
5.Atypical lymphocytes on FNA - F - lymph nodes contain abnormal lymphocytes which may resemble Reed-Sternberg cells

29
Q

30.Amoebiasis which is true

1.Organism is Ngareri Fowleri
2.Invasion through mucosa and into peyers patches
3.20% of infected people have dysentery
4.Cysts release trophozoites secondary to pancreatic enzymes.
5.Release exotoxin and denude epithelium

A

Answer: 20% of infected people have dysentery. ?unsure where the number comes from, but probably true as more commonly asymptomatic

ROBBINS
Entamoeba histolytica.
Cysts are resistant to gastric acid, allowing them to pass through the stomach without harm, and colonize the epithelial surface of the colon and release trophozoites, ameboid forms that reproduced under anaerobic conditions.

Amebiasis affects the cecum and ascending colon most often.

Dysentery develops when the ameba attach to colonic epithelium, induced apoptosis, invade crypts, and burrow laterally into lamina propria.

Can penetrate splanchnic vessels and embolize to the liver to produce abscesses in 40% of patients with amebic dysentery.
If it persists, and can reach the lung and heart by direct extension.

Treatment with metronidazole.

30
Q

31.Echinococcus Granulosus, which is true?

1.20% infected have liver lesions
2.Myocardial involvement is by antigen cross reaction rather than direct invasion
3.Entry is by breach in skin and mucosal membranes
4.Initially cysts begin at microscopic level

A

4.Initially cysts begin at microscopic level

31.Echinococcus Granulosus, which is true? (JS)

1.20% infected have liver lesions - F - liver is the most common organ of involvement (73%), followed by lung (14%), peritoneum (12%), kidney (6%). Can also involve spleen, CNS, orbit, bone, bladder, thyroid, prostate and heart.

2.Myocardial involvement is by antigen cross reaction rather than direct invasion - F -

3.Entry is by breach in skin and mucosal membranes - F - caused by ingestion of tapeworm eggs in dog faeces which hatch in the duodenum

4.Initially cysts begin at microscopic level - T - Cysts begin at microscopic size and progressively increase in size (can reach 10cm in 5 years)

31
Q

32.Classic appearance of Mallory Weiss tear is

1.Linear at gastro-oesophageal junction
2.Linear with hiatus hernia
3.Undermined proximal mucosa
4.Circular at gastro-oesophageal junction

A

1.Linear at gastro-oesophageal junction

32.Classic appearance of Mallory Weiss tear is (JS)

1.Linear at gastro-oesophageal junction - T - Longitudinal tear at the GOJ, seen in alcoholics due to excessive vomiting and reflux of gastric contents.

2.Linear with hiatus hernia

3.Undermined proximal mucosa

4.Circular at gastro-oesophageal junction

32
Q

33.Stomach cancer, which is not true

1.Benign ulcers on lesser curvature at antrum
2.Alcohol is not a risk factor
3.Most common is adenocarcinoma/ then lymphoma then carcinoid then GIST
4.Early gastric cancer is confined to mucosa/ submucosa with negative perigastric nodes

A

4.Early gastric cancer is confined to mucosa/ submucosa with negative perigastric nodes

33.Stomach cancer, which is not true (JS, TW & GC)
1.Benign ulcers on lesser curvature at antrum - T – gastric ulcers are predominantly located along the lesser curvature.
2.Alcohol is not a risk factor - T - Risk factors include Cigarette smoking, Diet (nitrites, smoked or salted foods, lack of fruit and veg), Chronic gastritis, H. pylori, Partial gastrectomy, Gastric adenomas, Barrett oesophagus, Family history, HNPCC
3.Most common is adenocarcinoma/ then lymphoma then carcinoid then GIST - T - Carcinoma (90-95%), Lymphoma (4%), Carcinoid (3%), Malignant stromal tumours (2%)
4.Early gastric cancer is confined to mucosa/submucosa without positive perigastric nodes - F - Early gastric carcinoma is defined as a lesion confined to the mucosa or submucosa, regardless of the presence or absence of perigastric lymph node metastases.
Changed option 4 from “with negative nodes” to make it less ambiguous.

33
Q

34.In colonic polyps, which is true

1.Peutz jager is sporadic hamartomatous polyps
2.HNPCC not associated with adenoma
3.Most common type in adults is villous
4.Juvenile polyps occur in ileum

A

2.HNPCC not associated with adenoma
*LW: agree this is most likely true out of options; robbins states colonic malignancies not usually associated with pre exisiting adenomas.

34.In colonic polyps, which is true: (JS, GC & TW)
1.Peutz Jeger is sporadic hamartomatous polyps - F - Autosomal dominant syndrome (not sporadic) characterised by multiple hamartomatous polyps
2.HNPCC (Hereditary Nonpolyposis Colorectal Cancer) not associated with adenoma – T (not T anymore!) - HNPCC is characterised by familial carcinoma of the colon, affecting predominantly caecum and proximal colon, which DON’T arise within adenomatous polyps. Historically this is true, however, now shown to be due to a DNA mismatch repair gene, where there is either accelerated adenoma-Ca sequence or dysplasia in an area of flat mucosa. (Robbins). Like most sporadic colorectal carcinomas, Lynch syndrome carcinomas appear to evolve from adenomas, but when detected the adenomas tend to be larger, flatter, and more proximal. Adenoma-carcinoma sequence is thought to progress much more rapidly in Lynch syndrome (UpToDate).
3.Most common type in adults is villous - F - Tubular adenomas are the most common (90%) followed by tubulovillous (5-10%) then villous (1%)
4.Juvenille polyps occur in ileum F - Focal hamartomatous polyps found most frequently in the rectum in children younger than 5y

34
Q

35.Crohns disease is not associated with

1.Hydronephrosis
2.Carcinoma
3.Hip arthropathy
4.Sclerosing cholangitis
5.Sclerosing peritonitis

A

5.Sclerosing peritonitis

35.Crohns disease is not associated with (JS)

1.Hydronephrosis - F - renal manifestations include stones, hydronephrosis, amyloid, cystitis and fistulas
2.Carcinoma - F - in ileum and colon, 4-20 times increased risk compared to general population
3.Hip arthropathy - F - AVN due to steroid treatment; enteropathic arthropathy
4.Sclerosing cholangitis - F - hepatobiliary manifestations include fatty infiltration, abscess, gallstones, cholecystitis, sclerosis cholangitis and bile duct carcinoma
5.Sclerosing peritonitis - T - asscociated with peritoneal dialysis

35
Q

36.Choledochal cysts (T/F)

1.If jaundiced implies secondary stricture
2.Pain means pancreatitis
3.Primary abnormality is stricture with secondary proximal dilatation

A

36.Choledochal cysts T/F (TW) Robbins p887

1.If jaundiced implies secondary stricture - F - intermittent obstructive jaundice in 33-80% of patients. Uncommon cause of obstructive jaundice.

2.Pain means pancreatitis - F - chronic and intermittent abdominal pain appears to be most common presenting symptom (50-96%) in patients older than 2yo. Pancreatitis has been described in approx 20% of pts at diagnosis.

3.Primary abnormality is stricture with secondary proximal dilatation - F - may be congenital or acquired. Congenital - due to ductal plate malformation of large bile ducts. Anomalous junction of pancreatic duct and CBD proximal to duodenal papilla allows pancreatic enzyme refulx into CBD.

36
Q

37.Patient sent for liver ultrasound, mother has primary biliary cirrhosis- which is true

1.Patient has 50% chance of having it
2.20% chance based on epidemiology
3.More likely to have it, if father has it
4.Has same risk as general population

A

37.Patient sent for liver ultrasound, mother has primary biliary cirrhosis- T/F (JS) - dodgy question

1.Patient has 50% chance of having it - F
2.20% chance based on epidemiology - F
3.More likely to have it, if father has it - F
4.Has same risk as general population - F –(this was previously thought to be the correct answer) not an inherited disorder, thought to have an autoimmune aetiology, disease of middle aged women with female to male ratio of 6:1
UpToDate:
The prevalence of primary biliary cirrhosis in females with 1 affected member is estimated to be 1000x greater than that of the general population.
Incidence of PBC in Australia is 25 per million (ie 1 in 40000 = 0.003%). Thus the risk with an affected family member (ie 1000x greater) is 1 in 40, or 2.5% chance.

37
Q

38.Acute acalculous cholecystitis which is unlikely

1.Post partum
2.HIV positive
3.TPN
4.Amyloid
5.Post surgical

A

4.Amyloid

38.Acute acalculous cholecystitis which is unlikely (JS)

*LW:
1.Post partum - TRUE
2.HIV positive - TRUE
3.TPN - TRUE
4.Amyloid - False; not specifically listed in common risk factors, while all others are.
5.Post surgical - TRUE

38
Q

39.Staging hepatocellular carcinoma, portal nodes negative, next most likely site is

1.Bone
2.Lungs
3.Adrenal
4.Brain
5.Spleen

A

2.Lungs

39.Staging hepatocellular carcinoma, portal nodes negative, next most likely site is (JS)

1.Bone
2.Lungs - T - most common site of mets (followed by adrenal, lymph nodes and bone) (Dahnert)
3.Adrenal
4.Brain
5.Spleen

39
Q

40.Patient with ulcerative colitis with elevated CEA

1.Likely colon cancer
2.May be colon cancer, but also raised in ulcerative colitis active inflammation

A

2.May be colon cancer, but also raised in ulcerative colitis active inflammation

40.Patient with ulcerative colitis with elevated CEA (JS)

1.Likely colon cancer

2.May be colon cancer but also raised in UC active inflammation - T - produced in embryonic tissue of the gut, pancreas and liver. Elevated in 60-90% of colorectal, 50-80% of pancreatic and 25-50% of gastric and breast cancer. Also elevated in cirrhosis, hepatitis, UC, Crohns and in smokers. Lacks specificity and sensitivity for early cancer detection but can be used in prognosis and detection of recurrence.
3.Active ulcerative Colitis

40
Q

41.Patient with elevated CA 125

1.Colon cancer
2.Ovarian cancer
3.Any ovarian pathology

A

3.Any ovarian pathology

41.Patient with elevated CA 125 (JS)

1.Colon cancer F

2.Ovarian cancer T surface epithelial tumours

3.Any ovarian pathology- T - elevated in ovarian cancer, endometrial, cervical, pancreatic, bronchial and breast cancer.
PID, endometriosis and first trimester pregnancy
Non-gynaecological: hepatic cirrhosis. acute peritonitis (esp TB), pancreatitis.
hepatoma, benign ovarian disease,
.

41
Q

42.Amyloid, which is false

1.In TB is AA type
2.Causes Sago spleen and Lardaceous spleen
3.Seen in x% of patients with renal failure

A

3.Seen in x% of patients with renal failure

42.Amyloid, which is false (JS)

1.In TB is AA type - T - AA type is secondary amyloidosis - Crohns, JRA, Reiter syndrome, ank spond, familial Mediterranean fever, Sjogren, dermatomyositis, vasculitis, chronic osteomyelitis, TB, bronchiectasis, CF, SLE etc
2.Causes Sago spleen and Lardaceous spleen - T - Sago spleen refers to deposits of amyloid within splenic follicles producing tapioca-like granules. Lardaceous spleen is seen with fusion of the deposits giving rise to large maplike areas of amyloidosis
3.Seen in x% of patients with renal failure - F - although renal involvement is common, renal function compromise is rare (RG 2004;24(2):405-416). Get amyloid (joint deposition) if on hemodialysis (incidence 21% 2yr dialysis, 50% 4-7y, 90% 7-13y, 100% >13y).

42
Q

43.Risk of Caecal carcinoma :

  1. 1% synchronous
  2. 0.1% synchronous
  3. 1% synchronous & metachronous
  4. 39% in the caecum and ascending colon
  5. 10% synchronous
A

43.Sep03.39 Risk of Caecal carcinoma : (TW)

1.1% synchronous - rate of synchronous CRC is about 5%.
2.0.1% synchronous
3.1% synchronous & metachronous - metachronous carcinoma occured in 2% of total, so metachronous and synchronous would be about 0.1%
4.39% in the caecum and ascending colon
5.10% synchronous
Info from Japanese Journal of Clinical Oncology 2003

43
Q

44.What are risk factors for malignancy in a stomach ulcer ?

1.pernicious anaemia
2.Crohn’s disease
3.coeliac disease
4.H. Pylori

A

Answer: H.Pylori

Pathology outlines/Robbins
- Chronic H.pylori infection is recognized as the main cause of noncardia (distal) gastric cancer.

  • PUD does not impart an increased risk of gastric cancer, but patients who have had partial gastrectomies for PUD have a slightly higher risk of developing cancer in the residual gastric stump, possibly due to hypochlorhydria, bile reflux, and chronic gastritis.
44
Q

45.Crohn’s disease in counselling a sibling of risk

  1. 1% risk
  2. No increased risk
  3. 10% risk
A
  1. 10% risk

45.Crohn’s disease in counselling a sibling of risk (JS)

1.1% risk
2.No increased risk
3.10% risk - T - lifetime risk if either a parent or sibling is affected is 9% (Robbins)

45
Q

46.Megaloblastic anaemia is not caused by :

1.Pernicious anaemia
2.Crohn’s disease
3.Coeliac disease
4.Pancreatic calcification

A

4.Pancreatic calcification

46.Megaloblastic anaemia is not caused by : (JS)

1.Pernicious anaemia - T - results in Intrinsic factor deficiency and therefore Vitamin B12 deficiency
2.Crohn’s disease - T - Ileitis can cause impaired absorption of Vit B12
3.Coeliac disease - T - Causes malabsorption
4.Pancreatic calcification – F - CME

**LJS - although also caused by panc insufficiency (panc enzymes allow cleavage of B12 from binder to allow it to bind intrinsic factor for SB absorption)

***LW:
All of above are listed as causes, hopefully option 5 was not a cause.

46
Q

47.Sickle cell anaemia. An unusual feature would be

1.splenomegaly
2.splenic atrophy/ absent
3.fungal dacralitis
4.aplastic crises

A

3.fungal dacralitis

47.Sickle cell anaemia. An unusual feature would be (JS)

1.splenomegaly - T - splenomegaly is seen in children in the early phase of the disease
2.splenic atrophy/ absent - T - continued scarring causes progressive shrinkage - autosplenectomy
3.fungal dacralitis - F - dacrylitis is caused by vaso-occlusive crisis (hypoxic injury and infarction caused by sickling), which can be precipitated by infection
4.aplastic crises - T - refers to temporary cessation of bone marrow activity, usually triggered by parvovirus infection, with a rapid worsening of anaemia

47
Q

48.HNPCC – barium enema is performed to look for

1.colonic carcinoma
2.polyps

A

1.colonic carcinoma

48.HNPCC – barium enema is performed to look for (JS)

1.Colonic carcinoma - T - HNPCC (Lynch syndrome) an AD condition, characterised by familial carcinomas of the colon, predominantly affecting the caecum and proximal colon. However the cancer DOES arise in adenomatous polyps (HNPCC – same rate of adenomas as general population, but higher rate of malignant transformation).
2.polyps

48
Q

49.Coeliac disease. What is the least likely?

1.anti-gliadin antibodies
2.treatment prevents development of lymphoma

A

2.treatment prevents development of lymphoma

49.Coeliac disease. What is the least likely? (JS)

1.anti-gliadin antibodies - T - due to a sensitivity to gluten (gliadin)
2.treatment prevents development of lymphoma - F - there is a long term risk of malignant disease at 2 times the usual rate - intestinal lymphomas (including disproportionately high number of T-cell lymphomas) and GIT and breast cancers.

49
Q
  1. Polycythemia Rubra Vera and splenomagaly —least likely

1.CML
2.ALL
3.Budd-Chiari
4.Cirrhosis

A

Answer: ALL (least likely)

PATHOLOGYOUTLINES
Clinical features
- 20% have document venous thrombosis, such as hepatic or portal vein thrombosis (associated Budd-Chiari).

Outcomes
- 2% develop AML.
- risk of leukemic transformation (ALL) is rare, except in patients treated with cytotoxic regimens.

ROBBINS
- Hepatic vein thrombosis/Budd-chiari is associated with polycythemia vera.

Budd-chiari is a cause of hepatic cirrhosis.

50
Q

51.HCC with lymph node mets — commonest metastatic site ?

1.lung
2.pleura
3.bone
4.brain

A

1.lung

51.HCC with lymph node mets — commonest metastatic site ? (JS)

1.lung
2.pleura
3.bone
4.brain

51
Q

52.Child for Ultrasound whose mother has PBC. What is the likelihood of PBC in child:

1.Patient has 50% chance of having it
2.20% chance based on epidemiology
3.More likely to have it, if father has it
4.Has same risk as general population
5. Increased risk.

A

Answer: Increased risk.

ROBBINS
PBC primarily a disease of middle-aged women, with a female predominance of 9:1. Peak incidence is 40-50 years of age.

Family members of PVC patients have an increased risk for development of the disease.

Autoimmune disorder, trigger unknown.
Most characteristic lab finding is Anti-mitochondrial antibodies.

No % or number quoted on robbins, statdx, pathologyoutlines, radiopeida.

52
Q

53.With regards to Oesophageal Carcinoma which is FALSE

1.Barretts has x40 increased association of SCC

A

1.Barretts has x40 increased association of SCC

53
Q

54.What are risk factors for malignancy in a stomach ulcer ?

1.Occurs in area of gastric atrophy

A

1.Occurs in area of gastric atrophy

54
Q

55.Which of the following statements concerning gastrointestinal diseases is most correct?

1.The colon is the most common site for GIT lymphoma
2.Pseudomembranous colitis is due to toxins of clostridium difficile
3.Inflammatory pseudo-polyps are a characteristic feature of Crohn’s colitis
4.Apthous ulcers are a characteristic feature of ulcerative colitis
5.Juvenile polyps typically occur in the ascending colon.

A

2.Pseudomembranous colitis is due to toxins of clostridium difficile

55.Which of the following statements concerning gastrointestinal diseases is most correct? (JS)

1.The colon is the most common site for GIT lymphoma - F - Stomach (55-60%), small intestine (25-30%), proximal colon (10-15%) and distal colon (10%) with appendix and oesophagus rarely involved.
2.Pseudomembranous colitis is due to toxins of clostridium difficile - T - Caused by exotoxins A and B of C. difficle after antibiotic therapy
3.Inflammatory pseudo-polyps are a characteristic feature of Crohn’s colitis - F - associated with UC but can occur in Crohns
4.Apthous ulcers are a characteristic feature of ulcerative colitis - F - associated with Crohns
5.Juvenile polyps typically occur in the ascending colon. - F - hamartomatous malformations of mucosa, typically in the rectum

55
Q

56.Which is NOT a hamartomatous/non neoplastic POLYP

1.Adenomatous
2.Peutz-Jeghers Syndrome
3.Hyperplastic
4.Inflammatory
5.Juvenile

A

1.Adenomatous

56.Which is NOT a hamartomatous/non neoplastic POLYP (JS)

1.Adenomatous - T – is neoplastic
2.Peutz Jegher - F – hamartoma
3.Hyperplastic - F – small epithelial polyps which arise at any age but are usually discovered in 6th and 7th decades, these are non-neoplastic with no adenomatous change
4.Inflammatory - F –
5.Juvenile - F – hamartomatous proliferation of lamina propria

56
Q

57.32yo female. Presents for a barium enema. History of HNPCC. What is the referring physician looking for:

1.Inflammatory bowel disease
2.Colonic carcinoma
3.Hereditary neoplastic colonic polyposis
4.Strictures
5.Hamartoma

A

2.Colonic carcinoma

57.32yo female. Presents for a barium enema. History of HNPCC. What is the referring physician looking for: (JS)

1.Inflammatory bowel disease
2.Colonic carcinoma - T - HNPCC (Lynch syndrome) an AD condition, characterised by familial carcinomas of the colon, predominantly affecting the caecum and proximal colon.
3.Hereditary neoplastic colonic polyposis (tumours still pass through adenoma stage but this answer is incorrect)
4.Strictures
5.Hamartoma

HEREDITARY NONPOLYPOSIS COLORECTAL CANCER (HNPCC)
(Lynch Syndrome)
Autosomal dominant familial syndrome
Increased risk of colorectal cancer and extraintestinal cancer
Endometrial
GBM

57
Q

58.Which is least likely in Crohn’s

1.Renal calculi
2.Cholangiocarcinoma
3.Biliary cirrhosis
4.Sclerosing cholangitis
5.Gallstones

A

3.Biliary cirrhosis

58.Which is least likely in Crohn’s (JS)

1.Renal calculi - F - urolithiasis in 5-10%
2.Cholangiocarcinoma - F - can develop bile duct and GB carcinoma (due to PSC)
3.Primary biliary cirrhosis - T - develop primary sclerosing cholangitis (not PBC)
4.Sacroilitis - F - ankylosing spondylitis in 3-16%
5.GIT malignancy - F - 4-20 times increased risk of colonic adenocarcinoma

58
Q

59.Crohn’s colitis is not associated with

1.Fat wrapping
2.Aphthous ulcer
3.Cobblestone
4.Backwash ileitis
5.Fistulas

A

4.Backwash ileitis

59.Crohn’s colitis is not associated with (JS/SK)

1.Fat wrapping – F adherent creeping mesenteric fat
2.Aphthous ulcer – F punched out aphthous ulcers
3.Cobblestone – F mucosal cobblestoning due to ulcers & fissures with intervening normal mucosa
4.Backwash ileitis - T - ulcerative colitis
5.Fistulas - F

59
Q

60.GIST tumours – which is correct?

1.Malignant > benign
2.Histological subtype most important in prognosis
3.Occur mainly in stomach and small bowel
4.Grow as an epithelial tumour

A

3.Occur mainly in stomach and small bowel

60.GIST tumours – which is correct? (TW)
1.Malignant > benign
2.Histological subtype most important in prognosis
3.Occur mainly in stomach and small bowel
4.Grow as an epithelial tumour

Location
70% stomach
30% small intestine
Rare in oesophagus and colon
May rarely occur in the ommentum and mesentery
90% are benign
either spindle cell or epithelioid
The spindle cell morphology is present in 70%–80%

60
Q

61.H.Pylori which is least correct

1.G-ve spiral
2.Attaches to small bowel epithelium
3.In duodenal + gastric ulcers
4.Associated with MALT lymphoma

A

2.Attaches to epithelium

61.H.Pylori which is least correct

1.Gram negative spiral – T – spiral shaped, microaerophilic, gram negative bacterium.
2.Attaches to epithelium in the small and large intestine – F - adheres to gastric epithelium.
3.Role in gastric cancers – T – HP can cause chronic active gastritis and atrophic gastritis, early steps in the carcinogenesis sequence. Studies have shown a clear link between HP and gastric adenocarcinoma. Source of gastric cancer may not be from gastric epithelial cells themselves, but rather from bone-marrow derived cells that differentiate into gastric epithelial cells in the presence of HP (UTD).
4.Associated with MALT lymphoma – T – multiple studies have shown association, and MALToma remission following eradication of HP.

61
Q

62.Which is most correct?

1.Angiodysplasia most commonly affects the sigmoid colon
2.short segment Hirschsprung most common in females
3.Meckel’s diverticulum is present on the mesenteric side
4.complications of necrotising enterocolitis include a stricture
5.pseudopolyps are a feature of Crohn’s disease

A

4.complications of necrotising enterocolitis include a stricture

62.Which is most correct? (JS)

1.Angiodysplasia most commonly affects the sigmoid colon - F - most often seen in the caecum or right colon
2.short segment Hirschsprung most common in females - F - more common in boys
3.Meckel’s diverticulum is present on the mesenteric side - F - form on the antimesenteric side of the small bowel
4.complications of necrotising enterocolitis include a stricture - T - complications of NEC include short bowel syndrome, malabsorption, strictures and recurrence of disease
5.pseudopolyps are a feature of Crohn’s disease - F - can occur in Crohns but more typical of UC

62
Q

63.Patient has a peptic ulcer with increased serum gastrin levels, what would you scan for:

1.Pancreas and duodenum
2.Pancreas, duodenum and lymph nodes
3.Mass in duodenum

A

63.Patient has a peptic ulcer with increased serum gastrin levels, what would you scan for: (TW, GC, JS) Robbins p782

1.Pancreas and duodenum
2.Pancreas, duodenum and lymph nodes – T – although the majority of gastrinomas in ZES and MEN-1 are in the pancreas & 1st part of the duodenum, would presumably still search remainder of gastrinoma triangle, then nodes to assess for metastatic deposits.
3.Mass in duodenum – this would be highest yield place to search if only able to look at one area.

Gastrinoma triangle – region bounded by confluence of the cystic and common bile ducts superiorly, the 2nd and 3rd portions of the duodenum inferiorly, and the neck and body of the pancreas medially – 85% of gastrinomas.
In contrast to sporatic gastrinomas, which are usually solitary lesions, gastrinomas that occur in patients with ZES and MEN-1 are usually multiple, less than 5mm in size, and located in the proximal duodenum. In ZES most (75%) are in the 1st portion of the duodenum, 14% in the distal duodenum, 11% in jejunum.

63
Q

64.With regards to carcinoid tumour, which is least correct:

1.Equal incidence to small bowel adenocarcinoma
2.Benign
3.Yellow, tan, circumscribed
4.Desmoplastic reaction
5.Dec order of incidence: appendix, SI, rectum

A

2.Benign
*LW: agree
More recent robbins has jejunum and ileum as > 40%, and appendix < 25%, while stat Dx has 90% of jejunal-ileal carcinoids being in ileum (of total 45%), while appendix only 16%.
So although option 5 is not true, I think option 2 is more incorrect, especially based on older Robbins data, likely from which question was written.

64.With regards to carcinoid tumour, which is least correct: (JS/SK)

1.Equal incidence to small bowel adenocarcinoma - T - 50% of SI malignant tumours
2.Benign - F - all are potentially malignant
3.Yellow, tan, circumscribed - T - characteristic feature is solid, yellow-tan appearance
4.Desmoplastic reaction - T - firm tumours due to desmoplastic reaction
5.Dec order of incidence: appendix, SI, rectum – T/F - appendix > SI > rectum > stomach > colon (not according to Robbins 8e p788 table 17-5!!)

64
Q

65.Which of the following statements concerning gastrointestinal diseases is most correct:

1.The colon is the most common site for GIT lymphoma
2.Inflammatory pseudopolyps are a characteristic of Crohn’s colitis
3.Aphthous ulcers are a characteristic feature of ulcerative colitis
4.Juvenile polyps typically occur in the rectum

A

4.Juvenile polyps typically occur in the rectum

65.Which of the following statements concerning gastrointestinal diseases is most correct: (GC)

1.The colon is the most common site for GIT lymphoma F - stomach 50% > SB > colon > oesophagus; multicentric in 10-50%.
2.Inflammatory pseudopolyps are a characteristic of Crohn’colitis F - of UC; represent islands of regenerating mucosa.
3.Aphthous ulcers are a characteristic feature of ulcerative colitis F - of Crohn’s disease; focal mucosal ulcers resembling canker sores.
4.Juvenile polyps typically occur in the rectum T - hamartomatous proliferations; in general they occur singly and in the rectum. Usually large in kids under 5yo. (1-3cm), smaller in adults (called retention polyps). No malignant potential.

65
Q

66.HNPCC: barium enema performed to look for:

1.Colonic carcinoma
2.Polyps
3.Detect colonic hamartoma
4.Hereditary neoplastic colonic polyposis

A

1.Colonic carcinoma

66.HNPCC: barium enema performed to look for: (JS)

1.Colonic carcinoma - T - HNPCC (Lynch syndrome) an AD condition, characterised by familial carcinomas of the colon, predominantly affecting the caecum and proximal colon. These don’t arise in adenomatous polyps.
2.Polyps
3.Detect colonic hamartoma
4.Hereditary neoplastic colonic polyposis

66
Q

67.Benign gastric ulcer ?which feature is most likely:

1.Greater curvature
2.Heaped up edges
3.Radiating folds
4.Chronic atrophic gastritis -

A

Answer: Radiating folds

ROBBINS
- Classic Peptic ulcer is a round-oval sharply punched-out defect.
- mucosal margin may overhand the base slightly, but is usually level with the surrounding mucosa.
- Active ulcers may be lined by a thin layer of fibrinoid debris underlaid by a predominantly neutrophilic inflammatory infiltrate. Beneath this granulation tissue infiltrated with mononuclear leukocytes and a fibrous or collagenous scar forms the ulcer base.
- Scarring may involve the entire thickness of the wall and pucker the surrounding mucosa into folds that radiate outwards.

In contrast, heaped-up margins are more characteristic of cancers.

Peptic ulcer disease is a sequelae of H.pylori-Associated chronic gastritis.

Atrophic gastritis can be caused by both H.pylori-associated chronic gastritis as well as Autoimmune gastritis, however only H.pylori-associated gastritis is prone to Peptic ulcer disease.

67
Q

68.Coeliac, which is true:

1.Flattened mucosa, elongated glands
2.Less than 2 times the risk of cancer -

A

1.Flattened mucosa, elongated glands

68.Coeliac, which is true: (JS)

1.Flattened villli, elongated glands – T - crypts are elongated, hyperplastic and tortuous but the overall mucosal thickness remains the same

2.At least 3 times the risk of cancer - F - there is a long term risk of malignant disease at 2 times the usual rate - intestinal lymphomas (including disproportionately high number of T-cell lymphomas) and GIT and breast cancers.

68
Q

69.Crohn’s, associated findings, which is false:

1.PBC
2.Renal stones
3.Cancer

A

1.PBC

69.Crohn’s, associated findings, which is false: (JS)

1.Primary biliary cirrhosis - F - develop primary sclerosing cholangitis (not PBC)
2.Sacroilitis - T – seronegative spondyloarthropathy in 3-16%
3.Cholangiocarcinoma - T - can develop bile duct and GB carcinoma (due to PSC)
4.Renal calculi - T - urolithiasis in 5-10%
5.GIT malignancy - T - 4-20 times increased risk of colonic adenocarcinoma

69
Q

70.Gastric marginal zone lymphoma, MALT oma, which is false:

1.They are a form of B cell lymphoma
2.They are most common in middle aged adults
3.Early dissemination is typical
4.They are associated with Helicobacter gastritis
5.They are associated with chronic inflammation secondary to autoimmune disorders

A

3.Early dissemination is typical

70.Gastric marginal zone lymphoma, MALToma, which is false: (JS)

1.They are a form of B cell lymphoma
2.They are most common in middle aged adults
3.Early dissemination is typical - F - they remain localised for prolonged periods at sites of origin with systemic spread late in their course
4.They are associated with Helicobacter gastritis
5.They are associated with chronic inflammation secondary to autoimmune disorders

70
Q

71.Achalasia of the oesophagus, which is false:

1.Dilated vestibule
2.25% have a dilated oesophagus and colon with chagas
3.Oesophageal perforation common
4.Most oesophageal perforations are iatrogenic

A

71.Achalasia of the oesophagus, which is false: (JS)

1.Dilated vestibule - F - vestibule is the dilated region of the stomach just above the GOJ; in the region of the LOS which fails to relax in achalasia - beaked tapering at GOJ
2.25% have a dilated oesophagus and colon with Chagas – T -The digestive forms of the disease lead to megaesophagus and/or megacolon in approximately one third of chronic cases, of which 20-50% also present with an associated cardiopathy.
3.Oesophageal perforation is a complication – T - Pneumatic dilatation for achalasia carries a significant and recognized risk of esophageal perforation (5%).
4.Most oesophageal perforations are iatrogenic T –

71
Q

72.Features of scleroderma include:

1.Upper third of oesophagus is preserved
2.Colon involvement rare
3.GIT involvement is rare
4.Oesophageal perforation is common

A

1.Upper third of oesophagus is preserved

72.Features of scleroderma include: (JS)

1.Upper third of oesophagus is preserved T - causes atony and aperistalsis of the lower 2/3 of oesophagus with patulous LOS and GOR
2.Colon involvement rare –F - colon is involved in 40-50% with pseudosacculations, eventual loss of haustra, marked dilatation and stercoral ulceration from retained faecal material
3.GIT involvement is rare – F - third most common manifestation (after skin and Raynaud’s), occurs in 40-45%
4.Oesophageal perforation is common - F - not mentioned in Dahnert, Primer or Mayo clinic book

72
Q

73.H pylori: whole lot of facts about relationship in gastric ca and lymphoma (T/F):

1.10% asymptomatic
2.20% chronic gastritis
3.Not linked to gastric cancer and lymphoma
4.Previously called enterobacter lymphoma

A

73.H pylori: whole lot of facts about relationship in gastric ca and lymphoma T/F: (JS)

1.10% asymptomatic - F - colonisation rates increase with age, reaching 50% in asymptomatic adults over 50 years
2.20% chronic gastritis - F - H pylori is present in 90% of patients with chronic gastritis affecting the antrum
3.Not linked to gastric cancer and lymphoma - F - associated with gastric cancer and lymphoma (GC - H.pylori is the first bacterium classified as a carcinogen. MALToma: eradication of H.pylori with AB therapy often leads to regression of tumour cells, which seems to depend on cytokines secreted by H.pylori-specific T cells for teir growth and survival. H.pylori gastritis - chronic inflammation - gastric atrophy - intestinal metaplasia - dysplasia - adenoCa)
4.Previously called enterobacter lymphoma - F - can’t find this anywhere….

73
Q

74.Elderly man with large bowel narrowing, biopsy showed segmental fibrosis and chronic inflammation, most likely:

1.Ischemia
2.Vasculitis
3.Carcinoma

A

1.Ischemia

74.Elderly man with large bowel narrowing, biopsy showed segmental fibrosis and chronic inflammation, most likely: (TW)

1.Ischemia - T - submucosal chronic inflammation and fibrosis can lead to stricture (in chronic ischemia).
2.Vasculitis
3.Carcinoma

74
Q

75.Angiodysplasia, most likely:

1.Most common in sigmoid
2.Can’t be seen macroscopically because covered mucosa
3.Cause of 20% of significant lower GI bleeding, either as massive blood loss of chronic loss
4.Not associated with other GIT lesions or CVS abnormalities

A

3.Cause of 20% of significant lower GI bleeding, either as massive blood loss of chronic loss

75.Angiodysplasia, most likely: (JS)

1.Most common in sigmoid - F - most common in caecum and right colon
2.Can’t be seen macroscopically because covered mucosa - F - tortuous dilations of submucosal and mucosal blood vessels, separated from the intestinal lumen by only the vascular wall and a layer of attenuated epithelial cells
3.Cause of 20% of significant lower GI bleeding, either as massive blood loss of chronic loss - T - account for 20% of significant lower intestinal bleeding
4.Not associated with other GIT lesions or CVS abnormalities - F - associated with Meckel diverticulum & aortic stenosis (20%)

75
Q

76.Known (or previous) strongyloides infection with oedema of the mucosa of ascending colon, what does it mean:

1.Autoinfection in the immunocompromised
2.Not related – only infects duodenum and SB
3.Ischaemia due to vascular invasion

A

1.Autoinfection in the immunocompromised

SCS: Dahert. Strongyloides hyperinfection syndrome. Extensive tissue invasion in immunocompromised. Thickened colon wall (due to florid transmural granulomatous inflammatory colitis causes by invasive larvae)
- Usually it just infects Duodenum and Jejunum (oedematous, irregular mucosal folds), duodenal stricture (3/4th portion).
-autoinfection: larve matures as it transits, re- enters via penetrating intestinal mucosa/perianal skin ans repeating life cycle.
-Life cycle: penetrates skin/mucous membranes. Passes from submucosal
Sites via lymphatics + venous circulation to lung. Larvae ascends bronchi into trachea after invading airway. Swallowed thus gets into alimentary tract.

76.Known (or previous) strongyloides infection with oedema of the mucosa of ascending colon, what does it mean: (TW) Robbins p391 & 805

1.Autoinfection in the immunocompromised - T - in contrast to other helminthic parasites, can complete life cycle entirely within human host. Autoinfection limited by intact immune respone, however in immunocompromised - can give rise to potentially fatal infection. During autoinfection, the rhabditiform (non-infectious) mature into filariform larvae (infectious) within the GIT - these can then penetrate the perianal skin or colonic mucosa to complete the cycle of autoinfection.
2.Not related – only infects duodenum and SB - F, but prob true in immunocompetent host - filariform larvae of Strongyloides stercoralia found in soil or in other materials contaminated with human faeces. Larvae penetrate skin and migrate hematogenously to lungs where they penetrate the alveolar air sacs. Larvae then ascent tracheobronchial tree and are swallowed. Then mature into adult worms and burrow into mucosa of duodenum and jejunum. Adult worms may life for up to 5y. Female adulte produces eggs, from which develop non-infectious larvae within the lumen of the GIT (which then pass into your turds).
3.Ischaemia due to vascular invasion - F - oedema and inflammation secondary to invasion
Endoscopy can show: duodenum - oedema, brown discoloration of mucosa, erythrmatous spots, subepithelial haemorrhage, and megaduodenum; colon - loss of vascular pattern, oedema, apthous ulcers, erosions, serpiginous ulcerations, and xanthoma-like lesions; stomach - thickened folds and mucosal erosions.

76
Q

77.Oesophageal narrowing in patient with CML, which is most likely:

1.CML recurrence
2.Graft verses host disease
3.Thrombocytopenia or something

A

????

77
Q

78.20 yo with 20 colonic polyps, suspect FAP, which is most likely:

1.Need >100 polyps
2.Attenuated FAP
3.Polyps in Peutz Jegher restricted to SB
4.Normal patients…inheritance, sporadic mutations are rare
5.Thyroid cancer and mastoid osteoma in Gardners

A

2.Attenuated FAP

78.20 yo with 20 colonic polyps, suspect FAP, which is most likely: (TW)

1.Need >100 polyps - F - attenuated FAP can have <100; classic FAP needs > 100
2.Attenuated FAP - T - (UTD) attenuated from of FAP - affected patients have fewer than 100 colorectal adenomas and a delayed onset of colorectal cancer. Polyposis typically develops in 2nd or 3rd decate of life. Similarly high risk, but with older average age of cancer (54yo).
3.Polyps in Peutz Jegher restricted to SB - F - small intestine 64%, colon 64%, stomach 49%, rectum 32%.
4.Normal patients…inheritance, sporadic mutations are rare
5.Thyroid cancer and mastoid osteoma in Gardners - F - 20 colonic polyps. Gardners = FAP + benign manifestations: osteomas and dental abnormalities; cutenous lesions; desmoid tumors; congenital hypertrophy of th retinal pigment; adrenal adenomas; nasal angiofibromas. And malignant manifestations: extra-colonic malignancies duodenal; thyroid; pancreatic; gastric; CNS; hepatoblastoma; small bowel distal to duodenum; possibly adrenal.

78
Q

79.Adenocarcinoma distal oesophagus, most likely:
1.Barrett’s oesophagus

A

1.Barrett’s oesophagus

79
Q

80.HNPCC

1.Cancer arises in adenoma
2.Increased incidence of colonic polyps

A

1.Cancer arises in adenoma

80.HNPCC (TW, GC) – be wary of this question in regards to old evidence, vs current evidence.

1.Cancer arises in adenoma - T – CRCs in Lynch syndrome differ from typical CRCs in location, histology, and natural history. Like most sporadic CRCs, Lynch cancers appear to evolve from adenomas, but when detected the adenomas tend to be larger, flatter, and more often proximal, and more commonly have high-grade dysplasia and/or villous histology than sporadic adenomas.
2.Increased incidence of colonic polyps - F - adenomas do occur in low numbers but considerably earlier than in the general adult poulation.

HNPCC / Lynch syndrome is the most common of the inherited colon cancer susceptibility syndromes. Predisposes to colon cancer as well as a varitey of other cancers (uterine, ovarian, gastric, small bowel, upper GUT, skin, brain).

80
Q

81.Giardia Diagnosis

1.Stomach / Proximal bowel
2.Helminth
3.Normal commensal in colon
4.Common in neonatal ICU

A

1.Stomach / Proximal bowel

  1. Giardia Diagnosis (TW) Robbins p806

1.Stomach / Proximal bowel – T – probably best answer – G. lamblia attach to mucosal surface of duodenum and jejunum.
2.Helminth – F – flagellated protozoan parasite.
3.Normal commensal in colon - F
4.Common in neonatal ICU – F – especially common in areas where there are poor sanitary conditions and insufficient water treatment facilities. Hopefully not in NICU, unless feeing the bubs infected turds.

81
Q

82.Candida, least likely:

1.Immunocompromised on skin
2.Skin only
3.HIV
4.Oesophagus in HIV

A

82.Candida, least likely: (TW) ???

1.Immunocompromised on skin - T - skin lesions (in candidemia) tend to appear suddenly as clusters of painless pustules on an erythematous base; they can occur on any area of the body.
2.Skin only - T - candida also causes eczematoid lesions in moist areas of the skin. Candida intertrigo.
3.HIV - T - AID’s defining illness. Oropharyngeal candidiasis is the most common opporutnistic infection in persons infected with HIV.
4.Oesophagus in HIV - T - oesophageal candidiasis is most common in HIV infected patients, in which it is an AIDS-defining illness, and in patients with hematologic malignancies.

82
Q

83.Alcoholic and cachetic patients with Bacteroides –which is false?

1.Aspiration from mouth organism
2.Commensal from mouth
3.Aspiration from stomach bug
4.Bowel diverticulitis
5.Osteomyelitis

A

3.Aspiration from stomach bug

83.Alcoholic and cachetic patients with Bacteroides (TW/SK) – which is false?

1.Aspiration from mouth organism - T - major anaerobic bacteria implicated in human disease include organisms from genus Bacteroides, Fusobacterium, Porphyromonas, and Prevotella (all gram-neg bacilli), clostridium, actinomyces, eubacterium (gram positive bacilli), peptostreptococcus and peptococcus (gram pos cocci), and veillonella (gram neg coccus). Thes organisms are normally found in oral cavity and they may result in disease when aspirated. Therefore, conditions that favor the occurance of aspiration, eg impaired consciousness, seizure, stroke, drug ingestion, and alcoholism, or conditions that impair ability to clear aspirated secrtions favor development of pulmonary anaerobic infections.
2.Commensal from mouth – T - Oral bacteria include streptococci, lactobacilli, staphylococci and corynebacteria, with a great number of anaerobes, especially bacteroides. (http://textbookofbacteriology.net/normalflora_3.html)
3.Aspiration from stomach bug F (SK) (can be commensal in distal SB & colon)
4.Bowel diverticulitis T can be involved
5.Osteomyelitis T

Emedicine: Bacteroides species are anaerobic bacteria that are predominant components of the bacterial florae of mucous membranes[1] and are therefore a common cause of endogenous infections. Bacteroides infections can develop in all body sites, including the CNS, the head, the neck, the chest, the abdomen, the pelvis, the skin, and the soft tissues. Inadequate therapy against these anaerobic bacteria may lead to clinical failure.

83
Q

84.Non specific region of small bowel thickening on CT, young man, no Hx:

1.Previous treatment for seminoma
2.RA new onset
3.Rota virus

A

1.Previous treatment for seminoma

84.Non specific region of small bowel thickening on CT, young man, no Hx: (TW)

1.Previous treatment for seminoma - ?T - previous Sx, chemotherapy, radiotherapy to nodes. Treatment depends on stage of seminoma.
2.RA new onset
3.Rota virus

84
Q

85.Mid oesophageal stricture 40 yo female, most likely:

1.If adeno ca likely long segment Barretts
2.SCC only in <5% here
3.SCC there are 4 patterns: multifocal, luminal, polypoid, ulcer

A

1.If adeno ca likely long segment Barretts

85.Mid oesophageal stricture 40 yo female, most likely: (JS)

1.If adeno ca likely long segment Barretts - T - Barretts is a cause of mid-oesophageal strictures and results in adenocarcinoma (although it is usually distal)
2.SCC only in <5% here - F - for SCC, 20% in upper third, 50% middle third, 30% lower third
3.SCC there are 4 patterns: multifocal, luminal, polypoid, ulcer - F - three patterns - protruded (polypoid exophytic), flat (diffuse infiltrative) and excavated (ulceration)

85
Q

.86.bowel cancer risks
1.Peutz Jeghers
2.FAP nephew
3.5 villous polpys excised previously

A

86.bowel cancer risks (TW/SK)
1.Peutz Jeghers – T – non-neoplastic polyp (hamartoma). Has increased risk of pancreatic, breast, lung, ovary and uterine cancer, but not colon cancer (path notes).
SK – Robbins & UTD say increased risk of colon cancer.
*LW: confusingly; Robbins states that with respect to small bowel adenocarcinoma peutz jeghers syndrome is a risk factor. Then goes on to say under colorectal carcinoma section “ when GI adenocarcinoma occurs it arises from concomitant adenomatous lesions”.

2.FAP nephew – T – Autosomal dominant. Polyposis typically develops in 10-30yo. 100% malignant transformation by 20y after Dx; age at carcinomatous development usually 20-40yo. This option would be only be true if accounting for appropriate age spread (between siblings [parent of nephew] + age difference with nephew).

3.5 villous polpys excised previously – T – as is seen in colon cancer, increasing size of adenomas and the presence of villous features are both risk factors for the development of invasive carcinoma within an adenoma.

86
Q

87.how to DX Giardia (? False)
1.total villous atrophy in jejunum
2.oocyst in stool
3.subtotal villous atropy
4.something about aspirate

A

Answer: Immunoflorescent detection of cysts in stool samples.

ROBBINS
Giardia Lamblia
- most common parasitic pathogen in humans
- fecal-oral route
- as few as 10 cyst required to be infected.
- Cause decreased expression of brush-border enzymes including lactase.
- Cause microvillous damage and apoptosis of small intestinal epithelial cells.
- Giardia tophozoites can be identified in duodenal biopsies
- Villous blunting wtih increased numbers of intraepithelial lymphocytes and mixed lamina propria inflammatory infiltrates can develop in patients with heavy infection.

Infection is usually documented by immunofluorescent detection of cysts in stool samples.

87
Q

88.If you can only do a sigmoidoscope % cancers missed are
1.5-15%
2.15-25%
3.40-60%
4.80-90%

A

Probable old question.

Robbins: evenly distributed throughout the colon

88
Q

89.can’t read my own notes, something about laryngeal met, least useful/related
1.5-hydroxytrptamine
2.atypical stricture in bowel
3.XRT
4.coeilac

A

*LW:
Most common primaries to met to larynx = RCC and melanoma.
So maybe atypical stricture in bowel, as melanoma likes to met to small bowel?

Laryngeal carcinoma spreads locoregionally, distant mets = Lungs (66%), bone (22%), liver (10%), mediastinum, and bone marrow.

89.can’t read my own notes, something about laryngeal met, least useful/related
1.5-hydroxytrptamine = 5-HT = serotonin
2.atypical stricture in bowel
3.XRT
4.coeilac

89
Q

90.H. Pylori… (T/F)
1.Associated with gastritis but most are asymptomatic
2.Causes a duodenitis more than a gastritis
3.Affects 1-2 % of adults

A

90.H. Pylori (TW)
1.Associated with gastritis but most are asymptomatic – T – vast majority of infected individual are asymptomatic (Dahnert pg 839).
2.Causes a duodenitis more than a gastritis – F – Gastritis 75%. Causes duodenal ulcers. in the duodenum H Pylori are confined to foci of gastric metaplasia. Gastric metaplasia refers to the presence of gastric epithelium in the 1st portion of the duodenum. Metaplastic foci provide areas for HP colonization, and probably have a role in the development of duodenitis. HP is almost always accompanied by gastritis and the diagnosis should be suspect in its absence.
3.Affects 1-2 % of adults – F – Conservative estimates suggest 50% of the worlds population is affected. In developing nations – majority of kids infected before 10yo, prevalence of adults peaks at more than 80% before 50yo. Developed countries: infection in kids is usual, but becomes more common in adulthood, 50% in those >60yo.

H. pylori infection is the most common cause of chronic gastritis. The disease most often presents as a predominantly antral gastritis with high acid production, despite hypogastrinemia.

90
Q

91.No meconium, complete micro colon, which is least correct?
1.Long segment hirschsrpungs
2.CF
3.Functional immaturity of the colon
4.Ilieal atresia

A

*LW: my preferred answer for being least correct:
3.Functional immaturity of the colon – F – AKA meconium plug syndrome / small left colon syndrome. Functional immaturity of ganglion cells in colon causing functional obstruction of newborn colon. Small left colon with transition point at splenic flexure.

91.No meconium, complete micro colon, which is least correct? (TW)
1.Long segment hirschsrpungs – T – total colonic Hirschsprung – entire colon may be small.
2.CF – F – meconium ileus is presenting finding in 10% of CF and is virtually pathognomonic of the disease. Essentially all patients with MI have CF. Microcolon present due to non-use. Ileal meconium plugs.
3.Functional immaturity of the colon – F – AKA meconium plug syndrome / small left colon syndrome. Functional immaturity of ganglion cells in colon causing functional obstruction of newborn colon. Small left colon with transition point at splenic flexure.
4.Ilieal atresia – T – Often no findings to Dx ileal atresia from other causes of distal bowel obstruction, so require contrast enema. Get microcolon (however if atresia occurred late in fetal life, microcolon may not be present and colon may contain some meconium).

91
Q

92.Splenic flexure cancer involving ____ ( sorry wrote the wrong word down )
1.Extending to spleen = T4
2.Duke 3
3.Dukes B2

A

Splenic flexure cancer involving ____ ( sorry wrote the wrong word down )
1.Extending to spleen = T4: TRUE
2.Duke 3 ? meaning C = local lymph node involvement, with extension to / through muscularis propria.
3.Dukes B2 = penetrates through muscularis propria without nodal involvement.

*LW:
Dukes is not really used any more, only historical use - hence why this must be featuring in our exam of the future…….
stage A: limited to mucosa
stage B1: extending into muscularis propria but not penetrating through it; nodes not involved
stage B2: penetrating through muscularis propria; nodes not involved
stage C1: extending into muscularis propria but not penetrating through it; nodes involved
stage C2: penetrating through muscularis propria; nodes involved
stage D: distant metastatic spread

92
Q

93.With regards to oesophageal carcinoma, which is false:
1.Barretts has a 40x increased association with SCC

A

93.With regards to oesophageal carcinoma, which is false: (TW)
1.Barretts has a 40x increased association with SCC – F – adenocarcioma. 30-40x increased risk of development of adenocarcinoma cf general population (with >2cm or Barrett mucosa). Robbins.
BE – condition in which abnormal, intestinal-type epithelium called specialized intestinal metaplasia replaces the stratified squamous epithelium that normally lines the distal oesophagus. Condition develops as a result of chronic GOR and predisposes to the development of adenocardinoma.

93
Q

94.Which is not a hamartomatous / non neoplastic polyp:
1.Adenomatous
2.Peutz Jegher
3.Hyperplastic
4.Inflammatory
5.Juvenile

A

1.Adenomatous - T – is neoplastic

94.Which is not a hamartomatous / non neoplastic polyp: (JS)
1.Adenomatous - T – is neoplastic
2.Peutz Jegher - F – hamartoma
3.Hyperplastic - F – small epithelial polyps which arise at any age but are usually discovered in 6th and 7th decades, these are non-neoplastic with no adenomatous change
4.Inflammatory - F – SK = Results from chronic cycles of injury & healing (e.g. solitary rectal ulcer syndrome due to impaired relaxation of the anorectal sphincter)
5.Juvenile - F – hamartomatous proliferation of lamina propria

94
Q

95.32 year old female presents for a barium enema, history of HNPCC (Hereditary Non Polyposis Colon Cancer). What is the referring clinician looking for:
1.Inflammatory bowel disease
2.Colonic carcinoma
3.Hereditary neoplastic polyps
4.Strictures
5.Hamartoma

A

2.Colonic carcinoma

95.32 year old female presents for a barium enema, history of HNPCC (Hereditary Non Polyposis Colon Cancer). What is the referring clinician looking for: (JS) SK – old question, likely written before it because evident the cancers still arise from adenomas

1.Inflammatory bowel disease
2.Colonic carcinoma - T - HNPCC (Lynch syndrome) an AD condition, characterised by familial carcinomas of the colon, predominantly affecting the caecum and proximal colon. They arise in polyps, but have a higher rate of malignant transformation.
3.Hereditary neoplastic polyps
4.Strictures
5.Hamartoma

95
Q

96.What are the risk factors for malignancy in a stomach ulcer:
1.Occurs in an area of gastritis

A

96.What are the risk factors for malignancy in a stomach ulcer: (TW)
1.Occurs in an area of gastritis – T – malignant transformation is unknown with duodenal ulcers and is extremely rare with gastric ulcers. When it occurs, the possibility exists that a seemingly benign lesions was from the outset a deceptive, ulcerative, gastric carcinoma. In addition, the underlying tendency towards dysplasia and carcinoma relates primarily to the associated chronic gastritis, rather then to the ulcer per se (Robbins)
Path course: Benign ulcers are benign and do not transform to malignant ulcers. Malignant ulcers are carcinomas that have ulcerated.
Size does not differentiate a benign from malignant ulcer.
Heaping-up of margins is characteristic of malignancy.

96
Q

97.Regarding Crohn’s disease on CT. Which of the following is least likely:
1.Primary biliary cirrhosis
2.Sacroilitis
3.Cholangiocarcinoma
4.Renal calculi
5.GIT malignancy

A

1.Primary biliary cirrhosis - T - develop primary sclerosing cholangitis (not PBC)

97.Regarding Crohn’s disease on CT. Which of the following is least likely: (JS, SK)

1.Primary biliary cirrhosis - T - develop primary sclerosing cholangitis (not PBC)
2.Sacroilitis - F – seronegative spondyloarthropathy in 3-16%
3.Cholangiocarcinoma - F - can develop bile duct and GB carcinoma (due to PSC)
4.Renal calculi - F - urolithiasis in 5-10%
5.GIT malignancy - F - 4-20 times increased risk of colonic adenocarcinoma (not as high risk for CRC as UC)

97
Q

98.Crohn’s colitis is not associated with:
1.Fat wrapping
2.Aphthous ulcers
3.Cobblestone
4.Backwash ileitis
5.Fistulas

A

4.Backwash ileitis - F - relates to UC. In 10% of patients with severe pancolitis in UC, the distal ileum may develop mild mucosal inflammation. Appendix may be involved in both Crohn and UC.

98.Crohn’s colitis is not associated with: (TW)

1.Fat wrapping - T - creeping fat - in diseased bowel segments, the serosa is granular and dull gray, and often the mesenteric fat wraps around the bowel surface.
2.Aphthous ulcers - T - characteristic sign of early disease is focal mucosal ulcers resembling canker sores
3.Cobblestone - T - as intervening mucosa can be relatively spared, can get cobble stone appearance.
4.Backwash ileitis - F - relates to UC. In 10% of patients with severe pancolitis in UC, the distal ileum may develop mild mucosal inflammation. Appendix may be involved in both Crohn and UC.
5.Fistulas - T - narrow fissures develop between the folds of the mucosa, often penetrating deeply through bowel wall and leading to bowel adhesions. Further extension of fissures leads to fistula or sinus tract formation.

98
Q

99.GIST tumour – which is correct:
1.Malignant > benign
2.Histological subtype most important in prognosis
3.Occur mainly in stomach and small bowel
4.Grow as an epithelial tumour

A

3.Occur mainly in stomach and small bowel - T- stomach most common site (2/3). Small bowel (esp duodenum) next most common site. Can occur anywhere in GI tract, and rarely occurs in oesophagus (where leiomyoma is more common).

99.GIST tumour – which is correct: (TW)
1.Malignant > benign - F - No GIST can truly be labelled as benign. Most common mesenchymal tumor of GIT. It is widely accepted that terms “benign” or “malignant” should not be applied to GIST, since these terms are not clinically useful for patient management (UTD). Clinical behaviour of GISTs is variable. Approximately 50% of completely resected GISTs can be expected to recur within 5y of follow up. All GISTs are now regarded as potentially malignant.

2.Histological subtype most important in prognosis - F - histo falls into 3 categories: spindle cell type 70%; epithelioid type 20%; mixed type 10%. Prognosis however is influenced by tumor size, mitotic rate, tumor site (sm intesting worse than stomach), and the completeness of resection.

3.Occur mainly in stomach and small bowel - T- stomach most common site (2/3). Small bowel (esp duodenum) next most common site. Can occur anywhere in GI tract, and rarely occurs in oesophagus (where leiomyoma is more common).

4.Grow as an epithelial tumour - F - GISTs are the most common nonepithelial tumors of the GIT. Submucosal tumor of GIT derived from interstitial cells of Cajal (which normally regulate peristaltic activity) & express CD117 (KIT) and 2/3 also express CD34. GISTs are of mesenchymal origin and are not related to leiomyomas or leiomyosarcomas.

99
Q

100.H. Pylori, which is least correct:
1.Gram negative spiral
2.Attaches to epithelium in the small and large intestine
3.Role in gastric cancers
4.Associated with MALT lymphoma

A

2.Attaches to epithelium in the small and large intestine – F - adheres to gastric epithelium exclusively (it’s niche).

100.H. Pylori, which is least correct: (TW/SK)
1.Gram negative spiral – T – spiral shaped, microaerophilic, gram negative bacterium.
2.Attaches to epithelium in the small and large intestine – F - adheres to gastric epithelium exclusively (it’s niche).
3.Role in gastric cancers – T – HP can cause chronic active gastritis and atrophic gastritis, early steps in the carcinogenesis sequence. Studies have shown a clear link between HP and gastric adenocarcinoma. Source of gastric cancer may not be from gastric epithelial cells themselves, but rather from bone-marrow derived cells that differentiate into gastric epithelial cells in the presence of HP (UTD).
4.Associated with MALT lymphoma – T – multiple studies have shown association, and MALToma remission following eradication of HP.
H.Pylori : Bacterial urease hydrolyzes gastric luminal urea to form ammonia that helps neutralize gastric acid and form a protective cloud around organism. Spiral shape + flagella, and mucolytic enzymes facilitates its passage through mucus layer to gastric surface of epithelium. Attaches to epithelium via receptor-mediated adhesion.
Route of transmission remains unknown: fecal/oral or oral/oral (not fecal/fecal) seems most likely.
HP is present in most patients who have duodenal ulcer that is not related to NSAID use.

100
Q

101.Which is most correct:
1.Angiodysplasia most commonly affects the sigmoid colon
2.Short segment Hirshsprung most common in females
3.Meckel’s diverticulum is present on the mesenteric side
4.Complications of necrotizing enterocolitis include stricture
5.Pseudopolyps are a feature of Crohn’s disease

A

101.Which is most correct: (JS)
1.Angiodysplasia most commonly affects the sigmoid colon - F - most often seen in the caecum or right colon
2.short segment Hirschsprung most common in females - F - more common in boys
3.Meckel’s diverticulum is present on the mesenteric side - F - form on the antimesenteric side of the small bowel
4.complications of necrotising enterocolitis include a stricture - T - complications of NEC include short bowel syndrome, malabsorption, strictures and recurrence of disease
5.pseudopolyps are a feature of Crohn’s disease - F - can occur in Crohns but more typical of UC

101
Q

102.Which is least likely in Crohn’s:
1.Caseating granuloma

A

102.Which is least likely in Crohn’s: (–)
1.Caseating granuloma - F - non-caseating.

102
Q

103.Leiomyoma, least common site
1.Heart
2.Myocardium
3.Blood vessels, vascular smooth muscle
4.Oesophagus
5.Stomach
6.Uterus

A

2.Myocardium – F – smooth muscle is found in the walls of all the hollow organs of the body except the heart.

103.Leiomyoma, least common site: (TW)
1.Heart – T – leiomyomas can be intravascular and extend to the heart. Primary IVC leiomomyoma / uterine leiomyoma with intravascular extension.
2.Myocardium – F – smooth muscle is found in the walls of all the hollow organs of the body except the heart.
3.Blood vessels, vascular smooth muscle – T – see answer 1. Leiomyomas of the head and neck probably arise from vascular smooth muscle.
4.Oesophagus - T
5.Stomach - T
6.Uterus - T

Leiomyomas represent the most common gynecologic and uterine neoplasms. Occusionally occur with unusual growth patterns or in unusual locations (eg diffuse periotenal leiomyomatosis, intravenous leiomyomatosis, benign metastasizing leiomyomas, retroperitoneal leiomyomas, parasitic leiomyomas).

103
Q

104.Which is not a hamartomatous / non neoplastic polyp:
1.Adenomatous
2.Peutz Jegher
3.Hyperplastic
4.Inflammatory
5.Juvenile

A

1.Adenomatous - T – is neoplastic

104.Which is not a hamartomatous / non neoplastic polyp: (JS)

1.Adenomatous - T – is neoplastic
2.Peutz Jegher - F – hamartoma
3.Hyperplastic - F – small epithelial polyps which arise at any age but are usually discovered in 6th and 7th decades, these are non-neoplastic with no adenomatous change
4.Inflammatory - F – Results from chronic cycles of injury & healing (e.g. solitary rectal ulcer syndrome due to impaired relaxation of the anorectal sphincter)
5.Juvenile - F – hamartomatous proliferation of lamina propria

104
Q

105.32 year old female presents for a barium enema, history of HNPCC (Hereditary Non Polyposis Colon Cancer). What is the referring clinician looking for:
1.Inflammatory bowel disease
2.Colonic carcinoma
3.Hereditary neoplastic polyps
4.Strictures
5.Hamartoma

A

2.Colonic carcinoma - T - HNPCC (Lynch syndrome) an AD condition, characterised by familial carcinomas of the colon, predominantly affecting the caecum and proximal colon.

105.32 year old female presents for a barium enema, history of HNPCC (Hereditary Non Polyposis Colon Cancer). What is the referring clinician looking for: (JS)

1.Inflammatory bowel disease
2.Colonic carcinoma - T - HNPCC (Lynch syndrome) an AD condition, characterised by familial carcinomas of the colon, predominantly affecting the caecum and proximal colon.
3.Hereditary neoplastic polyps
4.Strictures
5.Hamartoma

105
Q

106.Crohn’s colitis is not associated with:
1.Fat wrapping
2.Aphthous ulcers
3.Cobblestone
4.Backwash ileitis
5.Fistulas

A

4.Backwash ileitis - F - relates to UC. In 10% of patients with severe pancolitis in UC, the distal ileum may develop mild mucosal inflammation. Appendix may be involved in both Crohn and UC.

106.Crohn’s colitis is not associated with: (TW)

1.Fat wrapping - T - creeping fat - in diseased bowel segments, the serosa is granular and dull gray, and often the mesenteric fat wraps around the bowel surface.
2.Aphthous ulcers - T - characteristic sign of early disease is focal mucosal ulcers resembling canker sores
3.Cobblestone - T - as intervening mucosa can be relatively spared, can get cobble stone appearance.
4.Backwash ileitis - F - relates to UC. In 10% of patients with severe pancolitis in UC, the distal ileum may develop mild mucosal inflammation. Appendix may be involved in both Crohn and UC.
5.Fistulas - T - narrow fissures develop between the folds of the mucosa, often penetrating deeply through bowel wall and leading to bowel adhesions. Further extension of fissures leads to fistula or sinus tract formation.

106
Q

107.GIST tumour – which is correct: (TW)
1.Malignant > benign
2.Histological subtype most important in prognosis
3.Occur mainly in stomach and small bowel
4.Grow as an epithelial tumour

A

107.GIST tumour – which is correct: (TW)
1.Malignant > benign - F - No GIST can truly be labelled as benign. Most common mesenchymal tumor of GIT. It is widely accepted that terms “benign” or “malignant” should not be applied to GIST, since these terms are not clinically useful for patient management (UTD). Clinical behaviour of GISTs is variable. Approximately 50% of completely resected GISTs can be expected to recur within 5y of follow up. All GISTs are now regarded as potentially malignant.
2.Histological subtype most important in prognosis - F - histo falls into 3 categories: spindle cell type 70%; epithelioid type 20%; mixed type 10%. Prognosis however is influenced by tumor size, mitotic rate, tumor site (sm intesting worse than stomach), and the completeness of resection.
3.Occur mainly in stomach and small bowel - T- stomach most common site (2/3). Small bowel (esp duodenum) next most common site. Can occur anywhere in GI tract, and rarely occurs in oesophagus (where leiomyoma is more common).
4.Grow as an epithelial tumour - F - GISTs are the most common nonepithelial tumors of the GIT. Submucosal tumor of GIT derived from interstitial cells of Cajal (which normally regulate peristaltic activity) & express CD117 (KIT) and 2/3 also express CD34. GISTs are of mesenchymal origin and are not related to leiomyomas or leiomyosarcomas.

107
Q

108.No meconium, complete microcolon; surgeon says that the appendix doesn’t have any ganglion cells, most likely cause was:
1.Long segment hirschsrpungs
2.CF
3.Ilieal atresia
4.Meconium plug

A

108.No meconium, complete microcolon; surgeon says that the appendix doesn’t have any ganglion cells, most likely cause was: (GC/SK)
1.Long segment Hirschsprungs (TCA) = absence of parasympathetic ganglia in Meissner and Auerbach plexus secondary to arrest of craniocaudal migration of neuroblasts along vagal trunks before 12wks GA. Most is short segment disease ; long segment disesase in 15%, total aganglionosis in 5%. [Dahnert]
2.CF
3.Ilieal atresia
4.Meconium plug

Although “aganglionosis of the appendix alone is not reliable in the diagnosis of TCA. Biopsies obtained in multiple sites of the small intestine, colon, and rectum cannot yet be omitted as part of the diagnostic procedure” (Journal of Pediatric Gastroenterology & Nutrition:
September 1998 - Volume 27 - Issue 3 - pp 353-354)

108
Q

109.Crohn’s, associated findings, which is false:
1.PBC
2.forms of IBD
3.Renal stones
4.Cancer

A

109.Crohn’s, associated findings, which is false: (JS)
1.Primary biliary cirrhosis - F - develop primary sclerosing cholangitis (not PBC)
2.Sacroilitis - T – seronegative spondyloarthropathy in 3-16%
3.Cholangiocarcinoma - T - can develop bile duct and GB carcinoma (due to PSC)
4.Renal calculi - T - urolithiasis in 5-10% (calcium oxalate)
5.GIT malignancy - T - 4-20 times increased risk of colonic adenocarcinoma

109
Q

110.Which is not associated with Hamartoma?
1.Peutz-jeghers.
2.Cronkhite-canada.
3.Cowdens syndrome.
4.Turcots.

A

4.Turcots. - T - rare variant of FAP with colonic polyposis (adenomas not hamartomas) and tumours of the CNS (MB & GBM)

110.Which is not associated with Hamartoma? (JS)
1.Peutz-jeghers. - F - AD syndrome characterised by multiple hamartomatous polyps within the GIT and melanotic mucosal and cutaneous lesions
2.Cronkhite-canada. - F – Robbins – non-hereditary, often develops in people > 50 yrs, hamartomatous polyps of stomach, SB & colorectum + nail/hair/skin changes
3.Cowdens syndrome. - F - “Multiple hamartoma syndrome” with increased risk of breast cancer
4.Turcots. - T - rare variant of FAP with colonic polyposis (adenomas not hamartomas) and tumours of the CNS (MB & GBM)

110
Q

111.Which is true?
1.The colon is the most common site for GIT lymphoma
2.Pseudomembranous colitis is associated with clostridium difficile.
3.Inflammatory pseudo-polyps are a characteristic feature of Crohn’s colitis
4.Apthous ulcers are a characteristic feature of ulcerative colitis
5.Juvenile polyps typically occur in the ascending colon.
6.Apthous ulcers and UC.
7.Psudopolyps and Crohns

A

2.Pseudomembranous colitis is due to toxins of clostridium difficile - T - Caused by exotoxins A and B of C. difficle after antibiotic therapy

111.Which is true? (JS)
1.The colon is the most common site for GIT lymphoma - F - Stomach (55-60%), small intestine (25-30%), proximal colon (10-15%) and distal colon (10%) with appendix and oesophagus rarely involved.
2.Pseudomembranous colitis is due to toxins of clostridium difficile - T - Caused by exotoxins A and B of C. difficle after antibiotic therapy
3.Inflammatory pseudo-polyps are a characteristic feature of Crohn’s colitis - F - associated with UC but can occur in Crohns
4.Apthous ulcers are a characteristic feature of ulcerative colitis - F - associated with Crohns
5.Juvenile polyps typically occur in the ascending colon. - F - hamartomatous malformations of mucosa, typically in the rectum

111
Q

112.Associations with Gardeners syndrome, which is false?
1.Desmoid tumour.
2.Retinal anomalies.
3.Brain tumours.
4.Polyps.

A

3.Brain tumours. - F - Turcot syndrome is a variant of FAP with polyposis and tumours of the CNS

112.Associations with Gardners syndrome, which is false? (JS)
1.Desmoid tumour. - T - Soft tissue tumours including sebaceous/epidermoid inclusion cysts, fibromas, desmoid tumours, mammary fibromatosis, keloid formation
2.Retinal anomalies. - T - hypertrophy of retinal pigment epithelium (doesn’t affect vision) (Path outlines)
3.Brain tumours. - F - Turcot syndrome is a variant of FAP with polyposis and tumours of the CNS
4.Polyps. - T - multiple colonic polyps

112
Q

113.Non-caseating granulomas found on rectal biopsy associated with ?
1.Crohns.
2.UC.
3.Coeliac.
4.Pseudomembranous colitis.

A

1.Crohns - T

113.Non-caseating granulomas found on rectal biopsy associated with (TW)
1.Crohns - T
2.UC.
3.Coeliac.
4.Pseudomembranous colitis.

113
Q

114.Which is not a risk factors for Gastric ca:
1.Blood group A.
2.Pernicious anaemia.
3.Leiomyoma.
4.H.Pylori.

A

3.Leiomyoma. F - but malignant degeneration occurs.

114.Which is not a risk factors for Gastric ca: (GC)
1.Blood group A. T - slightly increased risk.
2.Pernicious anaemia. T - 3-fold.
3.Leiomyoma. F - but malignant degeneration occurs.
4.H.Pylori. T - 2 patterns of involvement: (a) antral type with high acid production - high risk of duodenal ulcer, (b) pangastritis with multifocal mucosal atrophy, low acid secretion - increased risk Ca. I read somewhere that HP infection has a 5-fold increased risk…
[Path outlines, Robbins]

114
Q

116.Which is not a feature of Zollinger-ellison?
1.Hypervascular GI mass.
2.Gastrinoma.
3.Increased ulcers.
4.Increased lymphoma in Gastric wall.
5.Atrophic Gastritis.

A

1.Hypervascular GI mass. T
2.Gastrinoma. T - gastrin-secreting tumour from within the gastrinoma triangle 90% of the time. More common in the duodenum than pancreas.
3.Increased ulcers. T - multiple in 10%; solitary ulcer in 90%; recurrent or intractable; unusual locations (postbulbar).
4.Increased lymphoma in Gastric wall. F - complications include: malignant islet cell tumour (in 60%), liver mets with continued gastric secretion, perforated DU/jejunal ulcer, oesoph stricture with reflux, obstruction, GI bleeding, gastric carcinoids (esp. in MEN1).
5.Atrophic Gastritis. ?F
LW (AJL agrees)
ZE is an example of a hypertrophic gastropathy characterised by enlargement of the rugal folds due to epithelial hyperplasia, so seems FALSE.

Gastric carcnoid tumours may be classified as:
Type I: ECL cell hyperplasia, hypergastrinaemia, chronic atrophic gastritis +/- pernicious anaemia; generally benign disease. ie. suspect if polyps found in a pt with chronic atrophic gastritis.
Type II: seen in the hypergastrinaemic state of ZES in assocn with MEN1; also arise from ECL cells in the setting of hyperplasia; multicentric. ie. multiple masses in setting of diffuse gastric wall thickening.
Type III: sporadic tumours not assocd with hypergastrinaemia; large solitary, may show ulceration; more likely to metastasise.

[Robbins; eMedicine; CT Stomach RG 2003]

115
Q

115.Scleroderma which is not a feature?
1.Sclerodactyly.
2.Raynauds.
3.Oesophageal dilatation.
4.Divericulae on the mesenteric side of bowel
5.Barretts oesophagus in 30-40%.

A

4.Divericulae on the mesenteric side of bowel not a feature - F - pseudodiverticula in small bowel - asymmetric sacculations with squared tops and broad bases on mesenteric side (due to eccentric smooth muscle atrophy).
*LW: –> diverticula / sacullations most commonly on ANTI-mesenteric border.

115.Scleroderma which is not a feature? (TW/SK)
1.Sclerodactyly - T - “tapered fingers” = atrophy + resorption of soft tissues of fingertips + soft tissue calcifications.
2.Raynauds - T - Raynaud may precede other symptoms by months/years
3.Oesophageal dilatation. - T – Mayo- decreased peristalsis in the distal two-thirds of the esophagus, with an incompetent lower esophageal sphincter. Gastroesophageal reflux and changes of peptic esophagitis often are observed. Chronically, a lower esophageal stricture is often present
4.Divericulae on the mesenteric side of bowel not a feature - F - pseudodiverticula in small bowel - asymmetric sacculations with squared tops and broad bases on mesenteric side (due to eccentric smooth muscle atrophy).
5.Barretts oesophagus in 30-40%. - T - 30% incidence. UTD says prevalence of Barrett’s metaplasia in PPI-treated systemic sclerosis patients is similar to that in other patients with GOR (13% in one study).

116
Q

117.Which is true of Carcinoid?
1.Need liver mets for carcinoid syndrome.
2.10% occur in terminal ileum.
3.Arise from enterochromatin and Kulchitsky cells.
4.Hypervascular mass on CT in arterial phase.

A

3.Arise from enterochromaffin and Kulchitsky cells. T - GIT carcinoids arise from enterochromaffin cells in crypts of Lieberkuhn (= argentaffinomma due to affinity for silver stain). Kulchitsky cells are generally seen in the lung and give rise to KCC1-3 carcinoid tumours. [Dahnert]

117.Which is most true of Carcinoid? (GC)

1.Need liver mets for carcinoid syndrome. F - Elevated levels of 5-HIAA, a metabolite, are found in blood and urine. Normally liver deactivates vasoactive amines (serotonin, histamine, bradykinin, others) released from carcinoid tumors; clinical symptoms occur if liver metastases are present or if tumor venous blood flow bypasses the liver (ie. primary pulmonary or ovarian carcinoids). CS occurs in 1% with carcinoid tumors, 20% with widespread metastases. Symptoms: vasomotor disturbances (cutaneous flushes, cyanosis of face and anterior chest, intermittent hypertension), palpitations, intestinal hypermotility (nausea, vomiting, diarrhea, cramps); also asthmatic attacks with bronchospasm, fibrosclerosis of AV and tricuspid valves, elastotic sclerosis of mesenteric vessels causing ischemia, dermal sclerosis, hepatomegaly. [Path outlines]

2.10% occur in terminal ileum. F - > 40% occur in small bowel, esp. in the distal ileum. [Dahnert]

3.Arise from enterochromaffin and Kulchitsky cells. T - GIT carcinoids arise from enterochromaffin cells in crypts of Lieberkuhn (= argentaffinomma due to affinity for silver stain). Kulchitsky cells are generally seen in the lung and give rise to KCC1-3 carcinoid tumours. [Dahnert]

4.Hypervascular mass on CT in arterial phase. T - although small solitary or multifocal carcinoids are typically not identified on CT scans, and may be hard to see on MRI (best visualized on T1 post gad + fat sat, where they manifest as nodules or focal areas of mural thickening with moderately intense gadolinium enhancement. [GI carcinoids RG 2007]
Added “most” to Q. Changed option c from “enterochromatin”.

WJI: Though there are exceptions option 1. is basically true also and an often stated concept.

117
Q

118.32 year old female presents for a barium enema, history of HNPCC (Hereditary Non Polyposis Colon Cancer). What is the referring clinician looking for:
1.Inflammatory bowel disease
2.Colonic carcinoma
3.Hereditary neoplastic polyps
4.Strictures
5.Hamartoma

A

2.Colonic carcinoma -

118.32 year old female presents for a barium enema, history of HNPCC (Hereditary Non Polyposis Colon Cancer). What is the referring clinician looking for: (JS)
1.Inflammatory bowel disease
2.Colonic carcinoma - T - HNPCC (Lynch syndrome) an AD condition, characterised by familial carcinomas of the colon, predominantly affecting the caecum and proximal colon. These don’t arise in adenomatous polyps.
3.Hereditary neoplastic polyps
4.Strictures
5.Hamartoma

118
Q

119.GIST tumour – which is correct:
1.Malignant > benign
2.Histological subtype most important in prognosis
3.Occur mainly in stomach and small bowel
4.Grow as an epithelial tumour

A

3.Occur mainly in stomach and small bowel - T- stomach most common site (2/3). Small bowel (esp duodenum) next most common site. Can occur anywhere in GI tract, and rarely occurs in oesophagus (where leiomyoma is more common).

119.GIST tumour – which is correct: (TW)
1.Malignant > benign - F - No GIST can truly be labelled as benign. Most common mesenchymal tumor of GIT. It is widely accepted that terms “benign” or “malignant” should not be applied to GIST, since these terms are not clinically useful for patient management (UTD). Clinical behaviour of GISTs is variable. Approximately 50% of completely resected GISTs can be expected to recur within 5y of follow up. All GISTs are now regarded as potentially malignant.
2.Histological subtype most important in prognosis - F - histo falls into 3 categories: spindle cell type 70%; epithelioid type 20%; mixed type 10%. Prognosis however is influenced by tumor size, mitotic rate, tumor site (sm intesting worse than stomach), and the completeness of resection.
3.Occur mainly in stomach and small bowel - T- stomach most common site (2/3). Small bowel (esp duodenum) next most common site. Can occur anywhere in GI tract, and rarely occurs in oesophagus (where leiomyoma is more common).
4.Grow as an epithelial tumour - F - GISTs are the most common nonepithelial tumors of the GIT. Submucosal tumor of GIT derived from interstitial cells of Cajal (which normally regulate peristaltic activity) & express CD117 (KIT) and 2/3 also express CD34. GISTs are of mesenchymal origin and are not related to leiomyomas or leiomyosarcomas.

119
Q

120.Features of diverticular disease include
1.Multiple true diverticulae
2.Circular muscular hypertrophy
3.Sacro-ilieitis
4.Increased risk of colonic cancer

A

2.Circular muscular hypertrophy - T - important feature of colonic diverticula = hypertrophy of the circular layer of the muscularis propria.

120.Features of diverticular disease include (TW)
1.Multiple true diverticulae - F - acquired diverticula lack or have an attenuated muscularis propria (cf congenital diverticula / true diverticula which have all three layers of bowel wall). Tend to occur alon gthe taeniae coli. Colonic diverticula are thin walled and are composed of a flattened or atrophic mucosa, compressed submucosa and attenuated or totally absent muscularis propria.
2.Circular muscular hypertrophy - T - important feature of colonic diverticula = hypertrophy of the circular layer of the muscularis propria.
3.Sacro-ilieitis - F - IBD, seronegative arthritis.
4.Increased risk of colonic cancer - ?F - the natural Hx of diverticulosis: 70% asymptomatic; 5-15% diverticular bleeding; 15-25% diverticulitis (of this 15-25%, 3/4 are simple, and 1/4 are complicated ie. abscess, obstruction, perforation, fistula). UTD says there may be a relation between diverticulosis and colon cancer. One series of 7000 patients found an excess number of colon and rectal cancers in the first 2 yrs after the Dx of diverticular disease, but not with more prolonged follow-up. There was, however, a long-term excess of left-sided colon cancers, suggesting a possible relation between these tumors and diverticular disease.

120
Q

121.Which of the following is not associated with ulcerative colitis:
1.Crypt abscess
2.Continuity
3.Cobblestone pattern
4.Primary sclerosing cholangitis
5.Uveitis

A

3.cobblestone pattern F - characteristic of Crohn’s (linear ulcers, relatively spared intervening mucosa develops a coarse cobbletstone appearance). UC: pseudopolyps (islands of regenerating mucosa), usually normal wall thickness & normal serosa.

121.Which of the following is not associated with ulcerative colitis: (GC)
1.crypt abscess T - neutrophils in glandular lumen. Image: crypt abscess causing mucosal stippling.
2.continuity T - diffuse distribution from rectum up, cf. skip lesions in Crohn’s.
3.cobblestone pattern F - characteristic of Crohn’s (linear ulcers, relatively spared intervening mucosa develops a coarse cobbletstone appearance). UC: pseudopolyps (islands of regenerating mucosa), usually normal wall thickness & normal serosa.
4.primary sclerosing cholangitis T - feature of UC (50-74%) and CD (13%).
5.uveitis T - feature of both UC and CD.

Diffuse disease limited to colon, rectal involvement with continuous proximal involvement; no skip lesions; no deep fissural ulcers; no transmural sinus tracts, no transmural lymphoid aggregates or granulomas.
Extraintestinal manifestations: migratory polyarthritis, sacroiliitis, ankylosing spondylitis, pyoderma gangrenosum, clubbing of fingertips, primary sclerosing cholangitis, pericholangitis, uveitis, cholangiocarcinoma (rare). [Path outlines; Adelaide notes]

121
Q

122.Which of the following is not associated with gastric carcinoma:
1.leiomyoma
2.Helicobacter pylori
3.Pernicious anaemia
4.Adenomatous polyps
5.Blood type A

A

1.Leiomyoma F - but malignant degeneration occurs.

122.Which of the following is not associated with gastric carcinoma: (GC)
1.Leiomyoma F - but malignant degeneration occurs.
2.Helicobacter pylori T - 2 patterns of involvement: (a) antral type with high acid production - high risk of duodenal ulcer, (b) pangastritis with multifocal mucosal atrophy, low acid secretion - increased risk Ca. I read somewhere that HP infection has a 5-fold increased risk…
3.Pernicious anaemia T - 3-fold.
4.Adenomatous polyps T - esp. villous adenomas; contain proliferative dysplastic epithelium; risk of harbouring adenoCa increases with size of polyp; synchronous Ca in 40% at time of diagnosis.
5.Blood type A T - slightly increased risk.
[Path outlines, Robbins]

122
Q

123.Which of the following is not a feature of Zollinger-Ellison syndrome:
1.Hypervascular pancreatic mass
2.gastrinoma
3.multiple peptic ulcers
4.atrophic gastritis
5.lymphoma of gastric mucosa

A

123.Which of the following is not a feature of Zollinger-Ellison syndrome: (GC)
1.Hypervascular pancreatic mass T
2.gastrinoma T - non-beta cell gastrin-secreting tumour of the pancreas
3.multiple peptic ulcers T - in 10%; solitary ulcer in 90%; recurrent or intractable; unusual locations (postbulbar).
4.atrophic gastritis F hyperplasia primarily of parietal cells in fundic glands; normal antral glands and gastric pits
5.lymphoma of gastric mucosa F - complications include: malignant islet cell tumour (in 60%), liver mets with continued gastric secretion, perforated DU/jejunal ulcer, oesoph stricture with reflux, obstruction, GI bleeding, gastric carcinoids (esp. in MEN1).

Gastric carcnoid tumours may be classified as:
Type I: ECL cell hyperplasia, hypergastrinaemia, chronic atrophic gastritis +/- pernicious anaemia; generally benign disease. ie. suspect if polyps found in a pt with chronic atrophic gastritis.
Type II: seen in the hypergastrinaemic state of ZES in assocn with MEN1; also arise from ECL cells in the setting of hyperplasia; multicentric. ie. multiple masses in setting of diffuse gastric wall thickening.
Type III: sporadic tumours not assocd with hypergastrinaemia; large solitary, may show ulceration; more likely to metastasise.
[Robbins; eMedicine; CT Stomach RG 2003]

123
Q

124.Ca pancreas risk factors

A

124.Ca pancreas risk factors

  • RF’s: smoking, high fat diet, chronic pancreatitis, diabetes, genetics (BRCA2, CDKN2A, Peutz-Jegher, PRSS1 & SPINK1)
124
Q

125.GIST tumour – which is correct:
1.Malignant > benign
2.Histological subtype most important in prognosis
3.Occur mainly in stomach and small bowel
4.Grow as an epithelial tumour

A

125.GIST tumour – which is correct: (GC)
1.Malignant > benign F - 90% benign
2.Histological subtype most important in prognosis F - may be subdivided according to IHC markers: tumours that show smooth m. differentiation (most common), those with neural differentiation (GANT), those with SM/neural dual differentiation, and tumours lacking differentiation. There seems to be no difference in the frequency of c-KIT (CD117) gene mutation among the various types of GISTs.
3.Occur mainly in stomach and small bowel T - 70% stomach, 30% small bowel.
4.Grow as an epithelial tumour F - most common mesenchymal tumour of GIT.
[Robbins; Adelaide notes]

125
Q

126.Angiodysplasia, most likely:
1.Most common in sigmoid
2.Can’t be seen macroscopically because covered mucosa
3.Cause of 20% of significant lower GI bleeding, either as massive blood loss of chronic loss
4.Not associated with other GIT lesions or CVS abnormalities

A

126.Angiodysplasia, most likely: (JS)
1.Most common in sigmoid - F - most common in caecum and right colon
2.Can’t be seen macroscopically because covered mucosa - F - tortuous dilations of submucosal and mucosal blood vessels, separated from the intestinal lumen by only the vascular wall and a layer of attenuated epithelial cells
3.Cause of 20% of significant lower GI bleeding, either as massive blood loss of chronic loss - T - account for 20% of significant lower intestinal bleeding
4.Not associated with other GIT lesions or CVS abnormalities - F - associated with aortic stenosis (20%) & Meckel diverticulum.

126
Q

127.Which of the following statements concerning gastrointestinal diseases is most correct:
1.The colon is the most common site for GIT lymphoma
2.Inflammatory pseudopolyps are a characteristic of Crohn’colitis
3.Aphthous ulcers are a characteristic feature of ulcerative colitis
4.Juvenile polyps typically occur in the rectum

A

4.Juvenile polyps typically occur in the rectum T - hamartomatous proliferations; in general they occur singly and in the rectum. Usually large in kids under 5yo. (1-3cm), smaller in adults (called retention polyps). No malignant potential.

127.Which of the following statements concerning gastrointestinal diseases is most correct: (GC)
1.The colon is the most common site for GIT lymphoma F - stomach 50% > SB > colon > oesophagus; multicentric in 10-50%.
2.Inflammatory pseudopolyps are a characteristic of Crohn’colitis F - of UC; represent islands of regenerating mucosa.
3.Aphthous ulcers are a characteristic feature of ulcerative colitis F - of Crohn’s disease; focal mucosal ulcers resembling canker sores.
4.Juvenile polyps typically occur in the rectum T - hamartomatous proliferations; in general they occur singly and in the rectum. Usually large in kids under 5yo. (1-3cm), smaller in adults (called retention polyps). No malignant potential.

127
Q

128.Which of the following is least frequently associated with malignancy:
1.tubular adenoma
2.Peutz-Jegher syndrome
3.Villous adenoma
4.Gardner’s syndrome
5.Turcot’s syndrome

A

1.Tubular adenoma F - all degrees of dysplasia may be encountered; risk of subsequent adenomas or CRC is related to size; higher risk if 6-10mm or larger, multiple adenomas or family history.

128.Which of the following is least frequently associated with malignancy: (GC)
1.Tubular adenoma F - all degrees of dysplasia may be encountered; risk of subsequent adenomas or CRC is related to size; higher risk if 6-10mm or larger, multiple adenomas or family history.
2.Peutz-Jegher syndrome T - AD disease with incomplete penetrance (instestinal polyposis + mucocutaneous pigmentation); increased risk of developing cancer of GIT, pancreas, breast, ovary, endometrium, testis; risk approaches 40% by 40yo.
3.Villous adenoma T - all degrees of dysplasia may be encountered; higher risk of malignancy than tubular adenoma; 40% risk if sessile and >4cm. Synchronous Ca is found in as many as 40% of these, the frequency being related to the size of the polyp.
4.Gardner’s syndrome T - AD disease (variant of FAP) with osteomas + soft tissue tumours. Malignant transformation of colonic polyps in 100%.
5.Turcot’s syndrome T - AR disease with colonic polyposis and CNS tumours (esp. supratentorial GBM, occ. medulloblastoma). Malignant transformation of colonic polyps in 100%; death from brain tumour in 2nd/3rd decade.
[Path outlines; Robbins; Dahnert]

128
Q

129.Which is not a feature of scleroderma:
1.Dilated atonic oesophagus with distal stricture
2.Multiple SB sacculation
3.Barrett’s oesophagus in 30-40%
4.Raynaud’s
5.Sclerodactyly

A

129.Which is not a feature of scleroderma (TW)
*LW: unsure of which one in this question!

1.Dilated atonic oesophagus with distal stricture – F?? - probably least correct. B&H: stiff dilated oesophagus that does not collapse with emptying, and wide gaping LES with free FOR. Despite free refulx, tight strictures of distal oesophagus are uncommon. Danhert says ‘fusiform stricture 4-5cm above GOJ from reflux oesophagitis”.
Primer lack of peristalsis in distal two-thirds with patulous GOJ unless stricture supervenes. (which occurs late secondary to GOR)
*LW: Robbins states associated dysfunction of lower 2/3 and LES gives rise to GORD and barrets metaplsia, and strictures…eventually destruction of wall leads to atony and dilation especially lower end. So I would favour this to be true.
**Dahnert: atonic/ hypotonic lower 2/3 oesophagus. Mild to moderate dilatation. But also says fusiform
Strictures secondary to GORD.

2.Multiple SB sacculation - T – pseudodiverticula/pseudosacculations along antimesenteric border, may involve SB and colon (due to eccentric smooth muscle atrophy). Mainly Tx/descending colon.

3.Barrett’s oesophagus in 30-40% - T?? - 30% incidence (where is this from?). UTD says prevalence of Barrett’s metaplasia in PPI-treated systemic sclerosis patients is similar to that in other patients with GOR (13% in one study).
*LW barretts in upto one third / 33% to TRUE.

4.Raynaud’s - T - Raynaud may precede other symptoms by months/years

5.Sclerodactyly - T - “tapered fingers” = atrophy + resorption of soft tissues of fingertips + soft tissue calcifications.

Scleroderma (Robbins p224 83) – GIT affected in 90%; progressive atrophy & collagenous fibrous replacement of the muscularis, most severe in oesophagus.

StatDx: “Best diagnostic clue: Dilated atonic esophagus with distal stricture”

129
Q

130.Which demonstrates non-caseating granuloma?
1.Ulcerative colitis
2.TB
3.Crohn’s disease
4.Pseudomembranous colitis
5.Ischaemic colitis

A

130.Which demonstrates non-caseating granuloma? (JS)
1.Ulcerative colitis - F -
2.TB - F - caseating granuloma
3.Crohn’s disease - T - sarcoid-like granulomas occur in about 50% of cases, absence of granulomas doesn’t exclude CD
4.Pseudomembranous colitis - F
5.Ischaemic colitis - F

130
Q

131.Which is not a cause of pneumatosis
1.CF
2.Asthma
3.FAP
4.COPD
5.Mesenteric ischaemia

A

*LW;
131.Which is not a cause of pneumatosis (TW)
1.CF: probably true (benign primary form)
2.Asthma: true (benign primary form)
3.FAP: i think less likely
4.COPD: true (primary benign form)
5.Mesenteric ischaemia: true secondary ischaemic form

Pneumatosis intestinalis - 2 forms: Primary form (15%) is a benign idiopathic condition in which multiple thin-walled cysts develop in the submucosa or subserosa of the colon. Primary form is often termed pneumatosis cystoides intestinalis. Secondary form (85%) is associated with obstructive pulmonary disease, as well as with obstructive and necrotic GI disease. Formation of pneumocysts results from the interaction of multiple factors, such as mucosal integrity, intraluminal pressure, bacterial flora, and intraluminal gas. Mucosal disruption may be caused by bowel wall trauma, obstruction, ischemia, or inflammation. Proposed theory for severe obstructive pulmonary disease is rupture of pulmonary blebs may cause air to dissect through the retroperitoneum, into the mesentery, and, finally to the bowel subserosa and submucosa.

131
Q

132.Ulcerative colitis is least associated with….
1.Crypt abscesses
2.Cobblestoning
3.Collar button ulcers
4.Malignancy
5.Sclerosing cholangitis

A

2- Cobblestoning is least associated with UC

132.Ulcerative colitis is least associated with…. (JS)
1.Crypt abscesses - Crohns and UC
2.Cobblestoning - Crohns
3.Collar button ulcers - UC
4.Malignancy - Higher risk in UC than Crohns
5.Sclerosing cholangitis - Higher association with UC than Crohns.

132
Q

133.Which is not associated with Zollinger Ellison syndrome?
1.Peptic ulcer disease
2.Lymphoma in the gastric folds
3.Atrophic gastritis
4.Hypervascular pancreatic mass
5.Gastrinoma

A

133.Which is not associated with Zollinger Ellison syndrome? (GC)
1.Peptic ulcer disease
2.Lymphoma in the gastric folds F - complications include: malignant islet cell tumour (in 60%), liver mets with continued gastric secretion, perforated DU/jejunal ulcer, oesoph stricture with reflux, obstruction, GI bleeding, gastric carcinoids (esp. in MEN1).
3.Atrophic gastritis F hyperplasia primarily of parietal cells in fundic glands; normal antral glands and gastric pits
4.Hypervascular pancreatic mass T
5.Gastrinoma T - non-beta cell gastrin-secreting tumour of the pancreas

Gastric carcnoid tumours may be classified as:
Type I: ECL cell hyperplasia, hypergastrinaemia, chronic atrophic gastritis +/- pernicious anaemia; generally benign disease. ie. suspect if polyps found in a pt with chronic atrophic gastritis.
Type II: seen in the hypergastrinaemic state of ZES in assocn with MEN1; also arise from ECL cells in the setting of hyperplasia; multicentric. ie. multiple masses in setting of diffuse gastric wall thickening.
Type III: sporadic tumours not assocd with hypergastrinaemia; large solitary, may show ulceration; more likely to metastasise.
[Robbins; eMedicine; CT Stomach RG 2003]

133
Q

134.Which is not a non-neoplastic polyp?
1.Hamartoma
2.Juvenile polyp
3.Hyperplastic polyp
4.Tubular adenoma

A

134.Which is not a non-neoplastic polyp? (JS)
1.Hamartoma
2.Juvenile polyp
3.Hyperplastic polyp
4.Tubular adenoma - T - a form of adenomatous polyp which result from epithelial proliferative dysplasia ie neoplastic epithelial lesion

134
Q

135.Which of the following has the lowest risk of malignant transformation?
1.Peutz Jegher hamartoma
2.Villous adenoma
3.Polyp in Gardner’s syndrome
4.Tubular adenoma
5.Polyp in Turcot’s syndrome

A

**LJS - seems like they are referring to the specific lesion, not the condition. PJ polyps are hamartomatous and not premalignant (despite the condition being associated with colon and other ca). With this wording, I would choose 1.
*LW, as always, agrees, especially given the word of transformation, felt this implies the lesion transforming to cancer

4.Tubular adenoma - F - cancer is rare in tubular adenomas less than 1cm but there is still a risk

135.Which of the following has the lowest risk of malignant transformation? (JS)
1.Peutz Jegher hamartoma - T - don’t have malignant potential themselves, but the patients are at increased risk of developing other cancers (colon, pancreas, breast, lung, ovary and uterus)
2.Villous adenoma - F - all adenomas arise due to epithelial dysplasia and therefore have malignant potential. Risk of cancer in villous adenomas >4cm is 40%
3.Polyp in Gardner’s syndrome - F - as for FAP, 100% progression to adenocarcinoma
4.Tubular adenoma - F - cancer is rare in tubular adenomas less than 1cm but there is still a risk
5.Polyp in Turcot’s syndrome - F - as for FAP, 100% progression to adenocarcinoma

135
Q

136.Which is characterised by non-caseating granulomas?
1.Eosinophilic granuloma
2.Crohns disease

A

136.Which is characterised by non-caseating granulomas? (TW)
1.Eosinophilic granuloma
2.Crohns disease - T

136
Q

137.Pneumatosis intestinalis- which is not a cause?
1.CF
2.Asthma
3.Familial polyposis
4.Mesenteric ischaemia
5.COPD

A

137.Pneumatosis intestinalis- which is not a cause? (TW)
1.CF - T
2.Asthma - T
3.Familial polyposis - F
4.Mesenteric ischaemia - T
5.COPD - T
Pneumatosis intestinalis - 2 forms: Primary form (15%) is a benign idiopathic condition in which multiple thin-walled cysts develop in the submucosa or subserosa of the colon. Primary form is often termed pneumatosis cystoides intestinalis. Secondary form (85%) is associated with obstructive pulmonary disease, as well as with obstructive and necrotic GI disease.
Formation of pneumocysts results from the interaction of multiple factors, such as mucosal integrity, intraluminal pressure, bacterial flora, and intraluminal gas. Mucosal disruption may be caused by bowel wall trauma, obstruction, ischemia, or inflammation. Proposed theory for severe obstructive pulmonary disease is rupture of pulmonary blebs may cause air to dissect through the retroperitoneum, into the mesentery, and, finally to the bowel subserosa and submucosa.

137
Q

138.Diverticular disease causes partial bowel obstruction due to…
1.Diverticulitis
2.Hypertrophy of the circular layer of muscularis propria
3.Vesicocolic fistula

A

138.Diverticular disease causes partial bowel obstruction due to… (TW)
1.Diverticulitis
2.Hypertrophy of the circular layer of muscularis propria - T - taeniae coli are also usually prominent. Constipation and diarrhoea can result from the hypertrophy.
3.Vesicocolic fistula

138
Q

139.Ectopic pancreas least likely location
1.Stomach
2.Colon
3.Ileum
4.Jejunum
5.Duodenum

A

139.Ectopic pancreas least likely location (TW)
1.Stomach
2.Colon - T
3.Ileum
4.Jejunum
5.Duodenum
May occur anywhere in the GIT. Most frequent locations are the stomach, duodenum or proximal part of small intestine.

139
Q

140.Crohn’s colitis is not associated with:
1.Fat wrapping
2.Aphthous ulcers
3.Cobblestone
4.Backwash ileitis
5.Fistulas

A

140.Crohn’s colitis is not associated with: (TW)
1.Fat wrapping - T - creeping fat - in diseased bowel segments, the serosa is granular and dull gray, and often the mesenteric fat wraps around the bowel surface.
2.Aphthous ulcers - T - characteristic sign of early disease is focal mucosal ulcers resembling canker sores
3.Cobblestone - T - as intervening mucosa can be relatively spared, can get cobble stone appearance.
4.Backwash ileitis - F - relates to UC. In 10% of patients with severe pancolitis in UC, the distal ileum may develop mild mucosal inflammation. Appendix may be involved in both Crohn and UC.
5.Fistulas - T - narrow fissures develop between the folds of the mucosa, often penetrating deeply through bowel wall and leading to bowel adhesions. Further extension of fissures leads to fistula or sinus tract formation.

140
Q

141.Which is least correct regarding scleroderma?
1.Involves lower oesophagus
2.Sacculations on mesenteric side of small bowel
3.Associated with Barrett’s oesophagus
4.Sclerodactyly

A

2.Sacculations on mesenteric side of small bowel - F - pseudodiverticula = asymmetric sacculations with squared tops and broad bases on antimesenteric side (due to eccentric smooth muscle atrophy)

141.Which is least correct regarding scleroderma? (TW)
1.Involves lower oesophagus - T - normal peristalsis above aortic arch (where striatued muscle is present), hypotonia / atony and hypokinesia / aperistalsis in lower 2/3rd. With a patulous LOS.
2.Sacculations on mesenteric side of small bowel - F - pseudodiverticula = asymmetric sacculations with squared tops and broad bases on antimesenteric side (due to eccentric smooth muscle atrophy)
3.Associated with Barrett’s oesophagus - T - Barretts is a complication of chronic GOR 70%. Erosions and superficial ulcers (from asymptomatic reflux oesophagitis: No protective oesophageal contraction). Complications peptic stricture, aspiration, Barrett oesohagus, adenocarcinoma. Barretts occurs in about 30% of patients.
4.Sclerodactyly - T - “tapered fingers” = sclerodactyly = atrophy and resorption of soft tissues of fingertips and soft-tissue calcifications.

Scleroderma encompasses a spectrum of related disorders, most of which share a characteristic clinical feature of skin thickening due to an excess of collagen fibers. Classification system takes into account the different potential complications, prognoses, and management strategies for these disorders.
Simplest division:
Localised scleroderma - Linear scleroderma (abnormalities of skin and subcut tissue often following dermatomal distribution and are found predominantly on 1 side of body; En coup de sabre (type of linear scleroderma = lesions look like sabre blow); Morphea (localized or genarlized - patches of sclerotic skin).
Systemic scleroderma - “Systemic sclerosis” (emphasizes frequent involvement of internal organs is generally the most important manifestation of these conditions) - Diffuse cutaneous SSc; Limited cutaneous SSc; SSc sine scleroderma (only internal organ involvement); Environmentally induced scleroderma; Overlap syndrome where SSc coexist with elements ofther rheumatic disorders.

141
Q

142.Rectal biopsy shows granulomas. Most likely?
1.Ulcerative colitis
2.Crohns disease

A

142.Rectal biopsy shows granulomas. Most likely? (TW)
1.Ulcerative colitis - F - involves rectum and extends proximally. No granulomas. Initially - mononuclear inflammatory infiltrate in lamina proria - may develop crypt abscesses. Get further destruction of mucosa leading to ulceration. Backwash ileitis. Pseudopolyps. Continuous proximal extension. Higher risk of malignancy cf Crohns.
2.Crohns disease - T - Present in about 50% cases (Robbins), sarcoid-like granulomas may be present in all tissue layers, both within areas of active disease and in uninvolved regions of the bowel. Granulomas have been documented throughout the GIT from mouth to rectum. UTD says granulomas can be present in up to 30%, however are not diagnostic of Crohns if approprate infections are not excluded (ie Yersinia spp. Bechet’s disease, TB, lymphoma). Creeping fat. String sign. Skip lesions. Aphthous ulcers. Linear ulcers. Cobblestone. Fistula or sinus tract formation.

142
Q

143.Which is not a non neoplastic polyp
1.Juvenille
2.Hyperplastic
3.Tubular
4.Harmatomatous
5.Inflammatory

A

143.Which is not a non neoplastic polyp (JS)
1.Juvenille - F - hamartomatous proliferation of lamina propria
2.Hyperplastic - F - small epithelial polyps which arise at any age but are usually discovered in the 6th and 7th decades, these are non-neoplastic with no adenomatous change. Thought to be secondary to decreased epithelial cell turnover & shedding.
3.Tubular - T - form of adenomatous polyp
4.Harmatomatous - F - hamartoma
5.Inflammatory - F - Results from chronic cycles of injury & healing (e.g. solitary rectal ulcer syndrome due to impaired relaxation of the anorectal sphincter)

143
Q

144.Inflammatory bowel disease is most closely associated with
1.Primary biliary cirrhosis
2.Caroli’s Disease
3.Primary sclerosing cholangitis
4.Biliary atresia

A

144.Inflammatory bowel disease is most closely associated with (TW)
1.Primary biliary cirrhosis - F - primary biliary cirrhosis is associated with autoimmune disorders (RA, Hashimoto’s, Sjorgren’s, scleroderma, membranous GN, coeliac disease).
2.Caroli’s Disease - F - Caroli’s associated with increased risk of cholangiocarcinoma. Also associated with medullary sponge kidney & ARPKD.
3.Primary sclerosing cholangitis - T - 50-70% of PSC have IBD (ulcerative colitis in the majority ~70%), and 4% of IBD have PSC. M>F 7:3. Mean age at diagnosis 40yo.
4.Biliary atresia - F

144
Q

145.Ulcerative colitis is least associated with
1.Crypt abscesses
2.Cobblestoning
3.Collar button ulcers
4.Malignancy
5.Sclerosing cholangitis

A

145.Ulcerative colitis is least associated with…. (JS + GC)
1.Crypt abscesses - F - collection of neutrophils in the crypt lumens
2.Cobblestoning - T - characteristic of Crohn’s (linear ulcers, relatively spared intervening mucosa develops a coarse cobbletstone appearance). UC: pseudopolyps (islands of regenerating mucosa), usually normal wall thickness & normal serosa.
3.Collar button ulcers - F - due to undermining of superficial ulcers
4.Malignancy - F - 20-30 fold increased risk of carcinoma
5.Sclerosing cholangitis - F - more common than in Crohns

145
Q

146.Which is not associated with Zollinger Ellison syndrome?
1.Peptic ulcer disease
2.Lymphoma in the gastric folds
3.Atrophic gastritis
4.Hypervascular pancreatic mass
5.Gastrinoma

A

146.Which is not associated with Zollinger Ellison syndrome? (JS, GC)
1.Peptic ulcer disease – T - multiple in 10%; solitary ulcer in 90%; recurrent or intractable; unusual locations (postbulbar).
2.Lymphoma in the gastric folds - F - complications include: malignant islet cell tumour (in 60%), liver mets with continued gastric secretion, perforated DU/jejunal ulcer, oesoph stricture with reflux, obstruction, GI bleeding, gastric carcinoids (esp. in MEN1).
3.Atrophic gastritis - F hyperplasia primarily of parietal cells in fundic glands; normal antral glands and gastric pits. Considered a “hypertrophic gastropathy” in Robbins.
4.Hypervascular pancreatic mass - T - may arise in the pancreas, peripancreatic region or wall of the duodenum
5.Gastrinoma - T - Cause of ZE in 90% of cases = non-beta cell gastrin-secreting tumour of the pancreas

146
Q

147.Which of the following has the lowest risk of malignant transformation?
1.Peutz Jegher hamartoma
2.Villous adenoma
3.Polyp in Gardner’s syndrome
4.Tubular adenoma
5.Polyp in Turcot’s syndrome

A

*LW:
if wording is malignant transformation, then lowest risk is Peutz jeghers with regards to their hamartomas.

147.Which of the following has the lowest risk of malignant transformation? (GC) – be wary of this question, re: polyp and malignant transformation, vs malignancy in general.
1.Peutz Jegher hamartoma - F (SK) - AD disease with incomplete penetrance (intertestinal polyposis and mucocutaneous pigmentation); increased risk of developing cancer of GIT, pancreas, breast, ovary, endometrium, testis; risk approaches 40% by 40yo. Polyps themselves do not have malignant potential - hence if this question was ‘least frequently associated with colonic malignancy’ this would be best answer.
2.Villous adenoma - T - all degress of dysplasia may be encountered; higher risk of malignancy than tubular adenoma; 40% risk if sessile and >4cm. Synchronous Ca is found in as many as 40% of these, the frequency being related to the size of the polyp.
3.Polyp in Gardner’s syndrome - T - AD disease (variant FAP) with osteomas + soft tissue tumors. Malignant transformation polyps 100%.
4.Tubular adenoma - T (GC) - all degrees of dysplasia may be encountered; risk of subsequent adenomas or CRC is related to size. <5mm 0.5%, 5-9mm 1%, 10-20mm 5-10%, >20mm 10-50%. Most adenomas are tubular (75%) and are usually <10mm in diameter.
5.Polyp in Turcot’s syndrome T - AR disease with colonic polyposis and CNS tumors (esp supratentoria GBM, occ medulloblastoma). Malignant transformation of colonic polypa in 100%.

147
Q

148.Rectal biopsy shows granulomas. Most likely?
1.Ulcerative colitis
2.Crohns disease

A

148.Rectal biopsy shows granulomas. Most likely? (JS)
1.Ulcerative colitis
2.Crohns disease - T - sarcoid-like granulomas occur in about 50% of cases, absence of granulomas doesn’t exclude CD

148
Q

149.Which association is true?
1.Acute pancreatitis with decreased calcium levels
2.Chronic pancreatitis with ARDS
3.Islet cell tumours with Zollinger Ellison
4.Gallstones and Phrygian

A

149.Which association is true? (JS + GC)
1.Acute pancreatitis with decreased calcium levels - T (SK) - hypocalcaemia from precipitation of Ca2+ in fat necrosis; poor prognostic sign if persistent. Hyper Ca2+ is a potential aetiologic factor in acute pancreatitis.
2.Chronic pancreatitis with ARDS - F - may be a complication of acute pancreatitis
3.Islet cell tumours with Zollinger Ellison - T - gastrinomas.
4.Gallstones and Phrygian cap - F - variant seen in 2-6% of population; may have focal adenomyoma at apex of inverted fundus.

149
Q

150.Pneumatosis intestinalis - which is not a cause?
1.CF
2.Colonoscopy
3.Familial polyposis
4.Mesenteric ischaemia
5.Chemotherapy

A

150.Pneumatosis intestinalis - which is not a cause? (GC)
1.CF - T - alveolar disruption
2.Colonoscopy - T - mucosal disruption
3.Familial polyposis - F
4.Mesenteric ischaemia - T - bowel necrosis
5.Chemotherapy - T - increased mucosal permeability
There are at least 58 causative factors, main groups:
1 Bowel necrosis/gangrene (gas-forming bacteria) - NEC, mesenteric thrombosis, neutropaenic colitis, sepsis, volvulus, emphysematous gastritis, caustic ingestion
2. Mucosal disruption (increased intestinal gas pressure) - GIT obstruction, trauma, infection/inflammation
3. Increased mucosal permeability (mural lymphoid tissue defects allowing bugs in) - GVHD, organ/BM transplantation, AIDS, steroids, chemo/rads, collagen vascular disease, intestinal bypass enteropathy, diabetes.
4. Pulmonary disease (alveolar disruption with dissection of air along interstitium into retroperitoneum) - COPD, asthma, CF, trauma, retching + vomiting.
[Dahnert pg 757]
Changed option b from “asthma” and option e from “COPD”

150
Q

151.Diverticular disease causes partial bowel obstruction due to
1.PR bleeding
2.Hypertrophy of the circular layer of muscularis propria
3.Vesicocolic fistula
4.Diverticulitis

A

151.Diverticular disease causes partial bowel obstruction due to… (GC, TW)
1.PR bleeding - F - note that bleeding from diverticulosis is usually from right colon; diverticulitis usually sigmoid.
2.Hypertrophy of the circular layer of muscularis propria - T - exaggerated peristalsis often induces muscular hypertrophy in affected segments, with unusually prominent taenia coli and circular muscle bundles.
3.Vesicocolic fistula - F - complication of diverticulitis (in 14% of cases), most common type of fistula (also colovaginal, coloenteric, colocutaneous).
4.Diverticulitis – F? - colonic obstruction occurs in 12% of cases of diverticulitis. Partial obstruction may occur from a circumferential or focally thickened and inflamed bowel wall, or an inflamed diverticulum with pericolic inflam change. Chronic inflammation can result in stricture formation, leading to luminal narrowing

151
Q

152.Ectopic pancreas least likely location:
1.Stomach
2.Colon
3.Ileum
4.Jejunum
5.Duodenum

A

152.Ectopic pancreas least likely location: (GC)
1.Stomach - T - greater curvature of antrum, 1-6cm from pylorus, pylorus.
2.Colon - F
3.Ileum - T - also in Meckel’s diverticulum
4.Jejunum - T - proximal jejunum
5.Duodenum - T - duodenal bulb
Smooth cone/nipple-shaped submucosal nodule 1-5cm in size; central umbilication representing orifice of filiform duct; may be multiple; asymptomatic. [Dahnert pg826]

StatDx:
Those identifiable at imaging are nearly always in stomach or duodenum
Remaining sites: Ileum, Meckel diverticulum, liver, biliary tract, spleen, omentum, mesentery, lung, mediastinum, fallopian tube, esophagus, colon
Mainly lies submucosally (73%); can be located in muscular layer (17%), or in subserosa (10%)

152
Q

153.Which of the following is not associated with ulcerative colitis
1.crypt abscess
2.continuity
3.cobblestone pattern
4.primary sclerosing cholangitis
5.Uveitis

A

153.Which of the following is not associated with ulcerative colitis (TW, GC)
1.crypt abscess - T - neutrophils in glandular lumen (histo picture of crypt abscess, barium fo collar button).
2.continuity - T - diffuse distribution from rectum back, compared with skip lesions of Crohn’s.
3.cobblestone pattern - F - Crohns. As intervening mucosa can be relatively spared, can get cobblestone appearance.
4.primary sclerosing cholangitis - T - feature of both UC (50-74%) and CD (13%)
5.uveitis - T - feature of both UC and CD

153
Q

154.Which of the following is least frequently associated with malignancy
1.tubular adenoma
2.Peutz-Jegher syndrome
3.Villous adenoma
4.Gardner’s syndrome
5.Turcot’s syndrome

A

*AJL - Villous is the Villain therefore tubular is the other one… (for those that can never remember it)

1.tubular adenoma - F - all degrees of dysplasia may be encountered; risk of subsequent adenomas or CRC is related to size. <5mm 0.5%, 5-9mm 1%, 10-20mm 5-10%, >20mm 10-50%. Most adenomas are tubular (75%) and are usually <10mm in diameter. Key malignancy risk depends on SIZE & tubular adenomas tend to be smaller (Robbins 8e).

154.Which of the following is least frequently associated with malignancy (TW, GC) - caution RE the wording!!
1.tubular adenoma - F - all degrees of dysplasia may be encountered; risk of subsequent adenomas or CRC is related to size. <5mm 0.5%, 5-9mm 1%, 10-20mm 5-10%, >20mm 10-50%. Most adenomas are tubular (75%) and are usually <10mm in diameter. Key malignancy risk depends on SIZE & tubular adenomas tend to be smaller (Robbins 8e).
2.Peutz-Jegher syndrome - T - AD disease with incomplete penetrance (intertestinal polyposis and mucocutaneous pigmentation); increased risk of developing cancer of GIT, pancreas, breast, ovary, endometrium, testis; risk approaches 40% by 40yo. Colonic polyps do not have malignant potential - hence if this question was ‘least frequently associated with colonic malignancy’ this would be best answer.
3.Villous adenoma - T - all degress of dysplasia may be encountered; higher risk of malignancy than tubular adenoma; 40% risk if sessile and >4cm. Synchronous Ca is found in as many as 40% of these, the frequency being related to the size of the polyp.
4.Gardner’s syndrome - T - AD disease (variant FAP) with osteomas + soft tissue tumors. Malignant transformation polyps 100%.
5.Turcot’s syndrome - T - AR disease with colonic polyposis and CNS tumors (esp supratentoria GBM, occ medulloblastoma). Malignant transformation of colonic polypa in 100%.

154
Q

155.Which of the following is not a feature of Zollinger-Ellison syndrome
1.Hypervascular pancreatic mass
2.Gastrinoma
3.multiple peptic ulcers
4.atrophic gastritis
5.lymphoma of gastric mucosa

A

155.Which of the following is not a feature of Zollinger-Ellison syndrome (TW, GC)
1.Hypervascular pancreatic mass - T - islet cell tumor
2.Gastrinoma - T - non-beta cell gastrin-secreting tumor of the pancreas
3.multiple peptic ulcers - T - multiple in 10%; solitary ulcer in 90%; recurrent or intractable; unusual locations (postbulbar)
4.atrophic gastritis - F - hypergastrinaemia causes gastric gland hyperplasia (mainly parietal cells in fundus)
5.lymphoma of gastric mucosa - ?F - complications include: malignant islet cell tumor (in 60%), liver mets with continued gastric secretion, perforated DU/jejunal ulcer, oesophageal stricture with reflux, obstruction, GI bleeding, gastric carcinoids (esp in MEN1)

155
Q

156.Which is not a feature of scleroderma
1.Dilated atonic oesophagus with distal stricture
2.Multiple SB sacculation
3.Barrett’s oesophagus in 30-40%
4.Raynaud’s
5.Sclerodactyly

A

156.Which is not a feature of scleroderma (JS + TW)
1.Dilated atonic oesophagus with distal stricture - ??F - Normal peristalsis above aortic arch (where striatued muscle is present), hypotonia / atony and hypokinesia / aperistalsis in lower 2/3rd. Patulous LOS. Despite free reflux, tight strictures of the distal oesophagus are uncommon (Brandt & Helms), but can occur late (Primer/Mayo).
2.Multiple SB sacculation - T - pseudodiverticula = asymmetric sacculations with squared tops and broad bases on antimesenteric side (due to eccentric smooth muscle atrophy)
3.Barrett’s oesophagus in 30-40% - ? T - Barretts is a complication of chronic GOR 70%. Erosions and superficial ulcers (from asymptomatic reflux oesophagitis: No protective oesophageal contraction). Complications peptic stricture, aspiration, Barrett oesohagus, adenocarcinoma. Barretts occurs in about 30% of patients (where’s this from?? (CCF - 40% have barrretts Stat Dx).
4.Raynaud’s - T – Raynaud phenomenon “invariably present” in scleroderma. CREST - calcinosis, raynaud’s, esophageal dysmotility, scleroderma and telangiectasia
5.Sclerodactyly - T - tapered fingers” = sclerodactyly = atrophy and resorption of soft tissues of fingertips and soft-tissue calcifications.

156
Q

157.The partial bowel obstruction due to diverticular disease of the colon is usually due to?
1.Diverticulitis
2.Hypertrophy of the circular layer of the muscularis propria
3.Inflammatory lymph node enlargement
4.Vesico-colic fistula
5.Pelvic inflammatory disease

A
  1. The partial bowel obstruction due to diverticular disease of the colon is usually due to?
    a. Diverticulitis ?T can cause partial obstruction and secondary strictures (SK). TW: colonic obstruction occurs in 12% of cases of diverticulitis. Partial obstruction may occur from a circumferential or focally thickened and inflamed bowel wall, or an inflamed diverticulum with pericolic inflam change. Chronic inflammation can result in stricture formation, leading to luminal narrowing
    b. Hypertrophy of the circular layer of the muscularis propria T exaggerated peristalsis often induces muscular hypertrophy in affected segments, with unusually prominent taenia coli and circular muscle bundles.
    c. Inflammatory lymph node enlargement
    d. Vesico-colic fistula complication of diverticulitis (in 14% of cases), most common type of fistula (also colovaginal, coloenteric, colocutaneous).
    e. Pelvic inflammatory disease
157
Q

158.Adenomatous polyps are NOT a feature of?
1.Familial polyposis
2.Gardner’s syndrome
3.Villous adenoma
4.Peutz-Jeghers syndrome
5.Turcot’s syndrome

A
  1. Adenomatous polyps are NOT a feature of?
    a. Familial polyposis T FAP
    b. Gardner’s syndrome T FAP + soft tissue tumours, osteomas, thyroid tumours, dental
    c. Villous adenoma T
    d. Peutz-Jeghers syndrome F hamartomatous polyps
    e. Turcot’s syndrome T FAP + CNS tumours (GBM or MB)
158
Q

159.Which of the following statements is TRUE?
1.Familial polyposis syndromes are autosomal recessive
2.Gardener’s syndrome shares the same genetic defect as Familial adenomatous polyposis
3.In Turcot’s syndrome osteomas are common
4.Duodenal adenomas are uncommon in familial adenomatous polyposis
5.Extra-intestinal tumours occur as part of familial adenomatous polyposis

A

b. Gardener’s syndrome shares the same genetic defect as Familial adenomatous polyposis ?T FAP + soft tissue tumours, osteomas, thyroid tumours, dental. APC gene. Specific APC mutations have been associated with the development of other manifestations of FAP and explain variants such as Gardner syndrome and Turcot syndrome.
*LW agree

  1. Which of the following statements is TRUE? (SK/JA)
    a. Familial polyposis syndromes are autosomal recessive F AD defect in APC gene (chr 5)

b. Gardener’s syndrome shares the same genetic defect as Familial adenomatous polyposis ?T FAP + soft tissue tumours, osteomas, thyroid tumours, dental. APC gene. Specific APC mutations have been associated with the development of other manifestations of FAP and explain variants such as Gardner syndrome and Turcot syndrome.

c. In Turcot’s syndrome osteomas are common F Gardners syndrome (Turcot = FAP + GBM/MB)

d. Duodenal adenomas are uncommon in familial adenomatous polyposis F (according to Mayo book found in small bowel & colon, with hyperplastic gastric polyps). Dahnert – hamartomas of stomach, adenomas of duodenum, periampullary carcinoma.

e. Extra-intestinal tumours occur as part of familial adenomatous polyposis ? T occurs in the specific variant sub-syndromes with specific APC mutations Gardner’s & Turcot’s.

Robbins:
FAP is associated with a variety of extra-intestinal manifestations including congenital hypertrophy of the retinal pigment epithelium, which can generally be detected at birth and can be an adjunct to early screening. Specific APC mutations have been associated with the development of other manifestations of FAP and explain variants such as Gardner syndrome and Turcot syndrome. In addition to intestinal polyps, Gardner syndrome families have osteomas of mandible, skull, and long bones, epidermal cysts, desmoid tumors, thyroid tumors, and dental abnormalities, including unerupted and supernumerary teeth. Turcot syndrome is rarer and characterized by intestinal adenomas and tumors of the central nervous system. Two thirds of patients with Turcot syndrome have APC gene mutations and develop medulloblastomas. The remaining one third have mutations in one of several genes involved in DNA repair and develop glioblastomas.
Some FAP patients without APC loss have mutations of the base-excision repair gene MUTYH.[112] The role of these genes in tumor development is discussed below. In addition, certain APC and MUTYH mutations are associated with attenuated forms of FAP, which are characterized by delayed polyp development, the presence of fewer than 100 adenomas, and the delayed appearance of colon cancer, often to ages of 50 or above

159
Q

160.Which of the following is most CORRECT?
1.Barret’s mucosa is characterised by metaplasia
2.Zenker’s diverticuli is an esophageal diverticulum
3.Scleroderma affects the upper third of the oesophagus
4.Meckel’s diverticulum occurs on the mesenteric side of the bowel
5.Zollinger-Ellison syndrome is characterised by gastric atrophy

A

a. Barrett’s mucosa is characterised by metaplasia T distal squamous mucosa is replaced by metaplastic columnar epithelium – mucin-secreting Goblet cells define intestinal metaplasia & are required for the diagnosis

  1. Which of the following is most CORRECT?
    a. Barrett’s mucosa is characterised by metaplasia T distal squamous mucosa is replaced by metaplastic columnar epithelium – mucin-secreting Goblet cells define intestinal metaplasia & are required for the diagnosis
    b. Zenker’s diverticuli is an oesohageal diverticulum ?F an outpouching of the mucosa through Killian’s triangle, an area of muscular weakness between the transverse fibers of the cricopharyngeus and the oblique fibers of the lower inferior constrictor (Dahnert – pharyngo-oesophageal diverticulum)
    c. Scleroderma affects the upper third of the oesophagus F distal two-thirds is atonic with a patulous LOS
    d. Meckel’s diverticulum occurs on the mesenteric side of the bowel F antimesenteric
    e. Zollinger-Ellison syndrome is characterised by gastric atrophy F hyperplasia of parietal cells results in mucosal thickening **LJS - marked gastric fold thickening

WJI: b. is less correct for 2 reasons. Zenkers are in the pharynx just above the oesophagus and they are false diverticuli so 1. Is most correct.

160
Q

161.Which of the following is a neoplastic polyp of the bowel?
1.Hamartomatous polyps
2.Tubular adenoma
3.Hyperplastic polyps
4.Juvenile polyps
5.Inflammatory polyps

A

b. Tubular adenoma

  1. Which of the following is a neoplastic polyp of the bowel?
    a. Hamartomatous polyps F
    b. Tubular adenoma
    c. Hyperplastic polyps F small <5mm epithelial smooth protrusions of mucosa – single or multiple, most commonly in left colon – composed of well formed glands & crypts lined by non neoplastic epithelial cells – no malignant potential
    d. Juvenile polyps F focal mucosal hamartoma in children < 5 yrs – may be sporadic or in association with rare juvenile polyposis syndrome – 80% occur in the rectum
    e. Inflammatory polyps F Results from chronic cycles of injury & healing (e.g. solitary rectal ulcer syndrome due to impaired relaxation of the anorectal sphincter)
161
Q

162.Which of the following is NOT a characteristic of ulcerative colitis
1.Crypt-abscesses
2.“Cobblestone” pattern
3.Continuity
4.Uveitis
5.Primary sclerosing cholangitis

A
  1. Which of the following is NOT a characteristic of ulcerative colitis
    a. Crypt-abscesses T Crypt inflammation (neutrophils) & abscesses with crypt architectural disarray
    b. “Cobblestone” pattern F seen in Crohn disease (intervening normal mucosa b/w ulcers & fissures)
    c. Continuity T
    d. Uveitis T
    e. Primary sclerosing cholangitis T
162
Q

163.Which of the following has the LEAST risk of malignancy transformation?
1.Tubular adenoma
2.Polyp in Peutz-Jegher’s syndrome
3.Villous adenoma
4.Adenoma in Gardner’s syndrome
5.Adenoma in Turcot’s syndrome

A

b. Polyp in Peutz-Jegher’s syndrome increased risk of bowel cancer but not from the hamartomatous polyp itself, increased risk of other cancers

  1. Which of the following has the LEAST risk of malignant transformation?
    a. Tubular adenoma
    b. Polyp in Peutz-Jegher’s syndrome increased risk of bowel cancer but not from the hamartomatous polyp itself, increased risk of other cancers
    c. Villous adenoma
    d. Adenoma in Gardner’s syndrome
    e. Adenoma in Turcot’s syndrome

WJI: another recall specifies adenomatous polyp in peutz jeghers in which case answer would be tubular adenoma.

163
Q

164.Concerning Zollinger-Ellison syndrome, which of the following is NOT a characteristic feature?
1.Hypervascular pancreatic mass
2.Gastrinoma
3.Lymphoma of gastric mucosal folds
4.Atrophic gastritis
5.Multiple peptic ulcers

A
  1. Concerning Zollinger-Ellison syndrome, which of the following is NOT a characteristic feature? (SK/GC)
    a. Hypervascular pancreatic mass - T - islet cell tumor
    b. Gastrinoma - T - non-beta cell gastrin-secreting tumor of the pancreas
    c. Lymphoma of gastric mucosal folds - ?F - complications include: malignant islet cell tumor (in 60%), liver mets with continued gastric secretion, perforated DU/jejunal ulcer, oesophageal stricture with reflux, obstruction, GI bleeding, gastric carcinoids (esp in MEN1)
    d. Atrophic gastritis F - hypergastrinaemia causes gastric gland hyperplasia (mainly parietal cells in fundus). SK – listed in Robbins under the heading “Hypertrophic gastropathies”, with no mention of atrophy or atrophic gastritis. All other references only talk of hypertrophic change in the stomach, not atrophy.
    e. Multiple peptic ulcers T

ZES = syndrome characterized by gastric mucosal thickening and multiple ulcers.
Cannot find info on increased lymphoma risk or atrophic gastritis – given this, neither would be a “characteristic” feature. If I had to choose one as most false, I would go option D (it’s the opposite of the truth!).

164
Q

165.Which of the following is NOT a recognised feature of scleroderma?
1.Dilated atonic oesophagus with a distal stricture
2.Wide mouthed sacculations or true diverticuli on the mesenteric side of the bowel
3.Development of Barrett’s oesophagus in 30-40%
4.Raynaud’s phenomenon
5.Sclerodactyly

A

*LW:
Radopedia states sacculations on mesenteric border.
–> to check this in Robbins - annoyingly no mention of anything in Robbins.
So out of all options I would say sacculations is least correct, and hopfully in correct recall / contentious about which side sacculations arise.
–> UTD states ANTI mesenteric, so this is my preferred option for being incorrect

  1. Which of the following is NOT a recognised feature of scleroderma?
    a. Dilated atonic oesophagus with a distal stricture T dilated atonic distal 2/3 with patulous LOS; in later stages can get distal stricture from GOR
    b. Wide mouthed sacculations or true diverticuli on the mesenteric side of the bowel F pseudodiverticuli/pseudosacculations on antimesenteric border (Primer/Mayo) – although other references say they occur on the mesenteric border!
    c. Development of Barrett’s oesophagus in 30-40% ?T (stats?)
    d. Raynaud’s phenomenon T
    e. Sclerodactyly T
    - pseudodiverticula in small bowel - asymmetric sacculations with squared tops and broad bases on ANTImesenteric side (due to eccentric smooth muscle atrophy).
    B&H: stiff dilated oesophagus that does not collapse with emptying, and wide gaping LES with free FOR. Despite free reflux, tight strictures of distal oesophagus are uncommon. Danhert says ‘fusiform stricture 4-5cm above GOJ from reflux oesophagitis
165
Q

166.Pneumatosis intestinalis is NOT a recognised feature of?
1.Cystic fibrosis
2.Asthma
3.Familial polyposis
4.COPD
5.Mesenteric ischaemia

A
  1. Pneumatosis intestinalis is NOT a recognised feature of?
    a. Cystic fibrosis
    b. Asthma
    c. Familial polyposis F
    d. COPD
    e. Mesenteric ischaemia
166
Q

167.Non-caseating granulomas are found on rectal biopsy, the MOST likely diagnosis is?
1.UC
2.TB
3.Pseudomembranous colitis
4.Ischaemic colitis
5.Crohn’s disease

A

e. Crohn’s disease T non caseating granulomas (50-60%) composed of epithelioid cells, a rim of lymphocytes, and multinucleated giant cells. Rectum involved in 15% of CD patients.

  1. Non-caseating granulomas are found on rectal biopsy, the MOST likely diagnosis is?
    a. UC F no granulomas
    b. TB F caseating granulomas
    c. Pseudomembranous colitis F Pseudomembrane plaque like adhesions of fibrinopurulent necrotic debris and mucus to damaged mucosa
    d. Ischaemic colitis F
    e. Crohn’s disease T non caseating granulomas (50-60%) composed of epithelioid cells, a rim of lymphocytes, and multinucleated giant cells. Rectum involved in 15% of CD patients.
167
Q

168.Inflammatory bowel disease is MOST commonly associated with?
1.Primary biliary cirrhosis
2.Caroli disease
3.Primary sclerosing cholangitis
4.Von Meyenburg complexes
5.Biliary atresia

A

c. Primary sclerosing cholangitis

  1. Inflammatory bowel disease is MOST commonly associated with?
    a. Primary biliary cirrhosis F a/w autoimmune diseases, AMA (antibodies)
    b. Caroli disease F a/w MSK, ARPKD, ADPKD and increased risk of cholangiocarcinoma
    c. Primary sclerosing cholangitis
    d. Von Meyenburg complexes F biliary hamartomas
    e. Biliary atresia F
168
Q

The partial bowel obstruction due to diverticular disease of the colon is usually due to?
a. Diverticulitis
b. Hypertrophy of the circular layer of the muscularis propria
c. Inflammatory lymph node enlargement
d. Vesico-colic fistula
e. Pelvic inflammatory disease

A
  1. Hypertrophy of the circular layer of the muscularis propria T exaggerated peristalsis often induces muscular hypertrophy in affected segments, with unusually prominent taenia coli and circular muscle bundles.
  2. The partial bowel obstruction due to diverticular disease of the colon is usually due to?
  3. Diverticulitis ?T can cause partial obstruction and secondary strictures (SK). TW: colonic obstruction occurs in 12% of cases of diverticulitis. Partial obstruction may occur from a circumferential or focally thickened and inflamed bowel wall, or an inflamed diverticulum with pericolic inflam change. Chronic inflammation can result in stricture formation, leading to luminal narrowing
  4. Hypertrophy of the circular layer of the muscularis propria T exaggerated peristalsis often induces muscular hypertrophy in affected segments, with unusually prominent taenia coli and circular muscle bundles.
  5. Inflammatory lymph node enlargement
  6. Vesico-colic fistula complication of diverticulitis (in 14% of cases), most common type of fistula (also colovaginal, coloenteric, colocutaneous).
  7. Pelvic inflammatory disease
169
Q
    1. Which of the following is most CORRECT?
  1. Barret’s mucosa is characterised by metaplasia
  2. Zenker’s diverticulum is an oesohageal diverticulum
  3. Scleroderma affects the upper third of the oesophagus
  4. Meckel’s diverticulum occurs on the mesenteric side of the bowel
  5. Zollinger-Ellison syndrome is characterised by gastric atrophy
A

b. Zenker’s diverticuli is an oesophageal diverticulum F an outpouching of the mucosa through Killian’s triangle, an area of muscular weakness between the transverse fibers of the cricopharyngeus and the oblique fibers of the lower inferior constrictor. Dahnert calls it a “pharyngo-oesophageal” diverticulum!
**LJS - actually hypopharynx that herniates out - Prometheus

  1. Which of the following is most CORRECT?
    a. Barret’s mucosa is characterised by metaplasia T distal squamous mucosa is replaced by metaplastic columnar epithelium – mucin-secreting Goblet cells define intestinal metaplasia & are required for the diagnosis
    b. Zenker’s diverticuli is an oesophageal diverticulum F an outpouching of the mucosa through Killian’s triangle, an area of muscular weakness between the transverse fibers of the cricopharyngeus and the oblique fibers of the lower inferior constrictor. Dahnert calls it a “pharyngo-oesophageal” diverticulum!
    c. Scleroderma affects the upper third of the oesophagus F distal two-thirds is atonic with a patulous LOS
    d. Meckel’s diverticulum occurs on the mesenteric side of the bowel F antimesenteric
    e. Zollinger-Ellison syndrome is characterised by gastric atrophy F hyperplasia of parietal cells results in mucosal thickening
170
Q

Which of the following is NOT a characteristic of ulcerative colitis
1.Crypt-abscesses
2.“Cobblestone” pattern
3.Continuity
4.Uveitis
5.Primary sclerosing cholangitis

A

b. “Cobblestone” pattern F seen in Crohn disease (intervening normal mucosa b/w ulcers & fissures) – equivalent in UC is pseudopolyps

  1. Which of the following is NOT a characteristic of ulcerative colitis
    a. Crypt-abscesses T Crypt inflammation (neutrophils) & abscesses with crypt architectural disarray
    b. “Cobblestone” pattern F seen in Crohn disease (intervening normal mucosa b/w ulcers & fissures) – equivalent in UC is pseudopolyps
    c. Continuity T
    d. Uveitis T
    e. Primary sclerosing cholangitis T
171
Q

171.16. Which of the following has the LEAST risk of malignant transformation?
a. Tubular adenoma
b. ADENOMA in Peutz-Jegher’s syndrome
c. Villous adenoma
d. Adenoma in Gardner’s syndrome
a. Adenoma in Turcot’s syndrome

A

a. Tubular adenoma T tend to be small and therefore have lower risk

  1. Which of the following has the LEAST risk of malignant transformation?
    a. Tubular adenoma T tend to be small and therefore have lower risk
    b. ADENOMA in Peutz-Jegher’s syndrome (not a hamartoma) T increased risk of GI carcinoma, which arise independently of the hamartomatous polyps. Routine surveillance of the GIT is recommended.
    c. Villous adenoma T often larger & sessile, therefore higher risk
    d. Adenoma in Gardner’s syndrome T except for their remarkable numbers, these growths are morphologically indistinguishable from sporadic adenomas
    e. Adenoma in Turcot’s syndrome T
172
Q

172.18. Pneumatosis intestinalis is NOT a recognised feature of?
1.Cystic fibrosis
2.Asthma
3.Familial polyposis
4.COPD
5.Mesenteric ischaemia

A

c. Familial polyposis

Pneumatosis intestinalis is NOT a recognised feature of?
a. Cystic fibrosis
b. Asthma
c. Familial polyposis
d. COPD
e. Mesenteric ischaemia

173
Q

Which of the following is not a non-neoplastic polyp?
1.Adenomatous polyp
2.Polyp in peutz-jegher syndrome
3.Hyperplastic polyp
4.Inflammatory polyp
5.Juvenile polyp

A

a. adenomatous polyp (is a neoplastic polyp therefore is not a non-neoplastic polyp)

Which of the following is not a non-neoplastic polyp?
a. adenomatous polyp
b. polyp in Peutz-Jegher syndrome hamartomatous polyp
c. hyperplastic polyp composed of well formed glands & crypts lined by non neoplastic epithelial cells – no malignant potential
d. inflammatory polyp chronic cycles of injury & healing; no malignant potential
e. juvenile polyp focal mucosal hamartoma in children > 5 years

174
Q
  1. GIST
    a) Most commonly small bowel
    b) Arise from the pacemaker cells
    c) Arise from smooth muscle
    d) There is an effective drug treatment
    e) Unpredictable behaviour
A

b) Arise from the pacemaker cells T arise from, or share a common stem cell with, the interstitial cells of Cajal (ICC), which are normally present in muscularis propria & serve as pacemaker for peristalsis

1) GIST
a) Most commonly small bowel F Stomach (70%) > small bowel (30%)&raquo_space; colon > peritoneum
b) Arise from the pacemaker cells T arise from, or share a common stem cell with, the interstitial cells of Cajal (ICC), which are normally present in muscularis propria & serve as pacemaker for peristalsis
c) Arise from smooth muscle F Leiomyoma (benign mesenchymal tumour arising from smooth muscle and connective tissue elements, less common than GIST in GIT except in oesophagus)
d) There is an effective drug treatment T/F Imatinib (Glivec) – tyrosine kinase antagonist – used if unresectable, recurrent or metastatic – however drug efficacy often limited by development of resistance secondary to c-KIT mutations
e) Unpredictable behaviour T/F All are regarded as potentially malignant. GISTs have a spectrum from small benign, typically incidentally diagnosed nodules to malignant tumors (sarcomas) at all sites of occurrence. Mitotic activity and tumor size are the most important gross and microscopic features in the assessment of malignancy. Typically, tumors smaller than 5 cm. with less than 5 mitoses per 50 high magnification fields have been considered benign, although intestinal tumors with such parameters have some unpredictability and a low metastatic rate. Tumors with mitotic activity over 5 per 50 high magnification fields often behave in a malignant manner with potential for metastasis. Tumors greater than 5 cm. with low mitotic activity are somewhat unpredictable. However, they seem to be much more favorable in the stomach than in the intestines.

175
Q
  1. UC
    a) Granulomas
    b) Terminal ileitis and dilatation
    c) Crypt abscesses
A

c) Crypt abscesses T Crypt inflammation (neutrophils) & abscesses with crypt architectural disarray (Crypt abscess found in both UC & CD)

2) UC
a) Granulomas F non-caseating granulomas found in Crohn disease
b) Terminal ileitis and dilatation T/F backwash ileitis with ulcerated but patulous terminal ileum (chronic stage)
c) Crypt abscesses T Crypt inflammation (neutrophils) & abscesses with crypt architectural disarray (Crypt abscess found in both UC & CD)

176
Q
  1. MALT (?MALToma) which is true/false
    a) Associated with H Pylori
    b) Is T Cell
    c) Treatment is not with antibiotics
    d) Translocation
A

3) MALT (?MALToma) which is true/false Robbins p786
a) Associated with H Pylori T
b) Is T Cell F – indolent extra-nodal marginal zone B-cell lymphoma
c) Treatment is not with antibiotics F “eradication of H.pylori infection with antibiotics induces durable remissions with low rates of recurrence in most patients” Robbins p786
d) Translocation T three tranlocations are a/w gastric MALToma, most commonly t(11;18)(q21;q21)

177
Q
  1. Achalasia
    a) Decreased basal tone of the LES
    b) Myenteric plexus is affected
    c) Normal Peristalsis in the oesophagus
    d) Is associated with increasing achalasia (??Bad recall)
A

b) Myenteric plexus (is affected) T Due to T cell mediated destruction or complete absence of myenteric ganglion cells in muscularis propria of lower third of esophagus (PathOutlines)

4) Achalasia (added bracketed parts)
a) Decreased basal tone (of the LES) F increased LES tone, with incomplete LES relaxation
b) Myenteric plexus (is affected) T Due to T cell mediated destruction or complete absence of myenteric ganglion cells in muscularis propria of lower third of esophagus (PathOutlines)
c) Normal Peristalsis (in the oesophagus) F aperistalsis of the oesophagus
d) Is associated with increasing achalasia (??Bad recall)

178
Q
  1. UC, which is not associated?
    a. Primary Biliary Cirrhosis
    b. Crypt-abscesses
    c. Continuity
    d. Uveitis
    e. Primary sclerosing cholangitis
A

a) Primary Biliary Cirrhosis F PSC

5) UC which is NOT associated
a) Primary Biliary Cirrhosis F PSC
b) Crypt-abscesses T Crypt inflammation (neutrophils) & abscesses with crypt architectural disarray
c) Continuity T
d) Uveitis T
e) Primary sclerosing cholangitis T

179
Q
  1. Small bowel adenocarcinoma (? Most common site)
    a) Duodenal
    b) Proximal Jejunum
    c) Distal Jejunum
    d) Proximal Ileum
    e) Distal Ileum
A

a) Duodenal 50% arise in the duodenum, 30% in the jejunum, and 20% in the ileum (Emedicine)

6) Small bowel adenocarcinoma (? Most common site)
a) Duodenal 50% arise in the duodenum, 30% in the jejunum, and 20% in the ileum (Emedicine)
b) Proximal Jejunum
c) Distal Jejunum
d) Proximal Ileum
e) Distal Ileum F favoured site for carcinoid tumours

Robbins p822: “…the small intestine, which accounts for 75% of the overall length of the GI tract, is an uncommon site for benign and malignant tumors. Among malignant small intestinal tumors, adenocarcinomas and carcinoid tumors have roughly equal incidence, followed by lymphomas and sarcomas.”

180
Q
  1. Diverticulitis with partial obstruction
    a) Diverticulitis
    b) Hypertrophy of the circular layer of the muscularis propria
    c) Inflammatory lymph node enlargement
    d) Vesico-colic fistula
    e) Pelvic inflammatory disease
A

7) Diverticulitis with partial obstruction
a) Circular muscle hypertrophy

Likely question… The partial bowel obstruction due to diverticular disease of the colon is usually due to?
a. Diverticulitis ?T can cause partial obstruction and secondary strictures (SK). TW: colonic obstruction occurs in 12% of cases of diverticulitis. Partial obstruction may occur from a circumferential or focally thickened and inflamed bowel wall, or an inflamed diverticulum with pericolic inflam change. Chronic inflammation can result in stricture formation, leading to luminal narrowing
b. Hypertrophy of the circular layer of the muscularis propria T exaggerated peristalsis often induces muscular hypertrophy in affected segments, with unusually prominent taenia coli and circular muscle bundles.
c. Inflammatory lymph node enlargement
d. Vesico-colic fistula complication of diverticulitis (in 14% of cases), most common type of fistula (also colovaginal, coloenteric, colocutaneous).
e. Pelvic inflammatory disease

181
Q
  1. With regards to Lynch’s syndrome (hereditary non polyposis colorectal carcinoma), which is MOST correct?
    a. Typically present with colon cancer at an older age than the general population.
    b. Right colonic cancers are most common.
    c. Autosomal recessive
    d. Only slight increase in incidence of ovarian cancer
    e. Not associated with microsatellite instability.
A

b. Right colonic cancers are most common. T colorectal cancers in Lynch syndrome are more commonly proximal; nearly 70% of first colorectal cancers in Lynch syndrome arise proximal to the splenic flexure whereas only about 40 to 50 percent of sporadic colorectal cancers are in this region

  1. With regards to Lynch’s syndrome (hereditary non polyposis colorectal carcinoma), which is MOST correct?
    a. Typically present with colon cancer at an older age than the general population. F younger age
    b. Right colonic cancers are most common. T colorectal cancers in Lynch syndrome are more commonly proximal; nearly 70% of first colorectal cancers in Lynch syndrome arise proximal to the splenic flexure whereas only about 40 to 50 percent of sporadic colorectal cancers are in this region
    c. Autosomal recessive F AD (MSH2 & MLH1 DNA mismatch repair genes most common)
    d. Only slight increase in incidence of ovarian cancer ?T smaller increased risk of ovarian cancer, but not as high risk as colon & endometrial cancer
    e. Not associated with microsatellite instability. F the majority relate to defects in DNA mismatch repair genes MSH2 & MLH1 which lead to microsatellite instability
182
Q

199.Regarding bowel pathology, which is MOST correct?
a. Increased risk of Hirschprung’s disease in Down syndrome.
b. Cathartic colon more commonly affects the descending and sigmoid colon.
c. Meckel’s diverticulum occur on the mesenteric side of the small bowel.
d. Turner’s have an increased risk of Adenocarcinoma.
e. Tubular adenomas have a higher risk of malignancy than villous adenomas.

A

f. Increased risk of Hirschprung’s disease in Down syndrome. T HD is associated with several chromosomal abnormalities and syndromes, such as Trisomy 21, cardiac disease (especially septal defects), congenital central hypoventilation syndrome (CCHS), MEN2, Bardet-Biedl syndrome (BBS), Waardenburg syndrome, Mowat-Wilson syndrome (MWS), and Smith-Lemli-Opitz syndrome (UTD).

  1. Regarding bowel pathology, which is MOST correct?
    f. Increased risk of Hirschprung’s disease in Down syndrome. T HD is associated with several chromosomal abnormalities and syndromes, such as Trisomy 21, cardiac disease (especially septal defects), congenital central hypoventilation syndrome (CCHS), MEN2, Bardet-Biedl syndrome (BBS), Waardenburg syndrome, Mowat-Wilson syndrome (MWS), and Smith-Lemli-Opitz syndrome (UTD).
    g. Cathartic colon more commonly affects the descending and sigmoid colon. F Cathartic colon is a rarely seen but severe manifestation of prolonged laxative use. It is characterized by dilation of the large bowel, with decreased or absent haustrations noted on plain abdominal films or barium enema. The changes are usually most marked in the right colon, but can affect the entire colon (UTD).
    h. Meckel’s diverticulum occur on the mesenteric side of the small bowel. F antimesenteric side, usually within 2 feet of ileocaecal valve
    i. Turner’s have an increased risk of Adenocarcinoma. F but have increased risk of gonadoblastoma (if have a Y chr), meningioma, childhood brain tumors, bladder, and uterine cancer (but not breast cancer) (UTD)
    j. Tubular adenomas have a higher risk of malignancy than villous adenomas. F now Robbins 8e just uses size, saying villous tumours are usually larger therefore higher risk “villous tumours are the villain”
    CCF - When controlled for size no cancer risk difference tubular vs villious, “Although villous architecture does contains foci of invasion more frequently than tubular adenomas, villous architecture alone does not increase cancer risk when poly size is considered” p808 Robbins 9Ed
183
Q

200.Which of the following is correct with regards to Peutz-Jehgers disease
a. More common associated with the colon than small bowel
b. Hyperplastic polyps
c. Adenomatous polyps
d. No increased risk of malignancy
e. Hamartomatous polyps

A

e. Hamartomatous polyps T

  1. Which of the following is correct with regards to Peutz-Jehgers disease
    a. More common associated with the colon than small bowel F SB > colon > stomach
    b. Hyperplastic polyps F
    c. Adenomatous polyps F
    d. No increased risk of malignancy F Polyps have no malignant potential themselves, however patients at increased risk of carcinomas of the colon, pancreas, breast, lung, ovary, uterus & testis
    e. Hamartomatous polyps T
184
Q

201.Which of the following bowel conditions are associated with mucosal membranes? (at colonoscopy)
a. Ischaemic colitis and Crohn’s disease
b. Pseudomembranous colitis and ischaemic colitis
c. Pseudomembranous colitis and infective colitis (dysentery)
d. Pseudomembranous colitis and Ulcerative colitis
e. Infective colitis (dysentery) and Ulcerative colitis

A

l. Pseudomembraneous colitis and ischaemic colitis T & T probably best answer
*LW: agrees

  1. Which of the following bowel conditions are associated with mucosal membranes? (at colonoscopy)
    k. Ischaemic colitis and Crohn’s disease T & F
    l. Pseudomembraneous colitis and ischaemic colitis T & T probably best answer
    m. Pseudomembraneous colitis and infective colitis (dysentry) T & T
    n. Pseudomembraneous colitis and Ulcerative colitis T & F
    o. Infective colitis (dysentery) and Ulcerative colitis T & F

Robbins p803 8e “pseudomembranes are not specific & may occur in PMC (CDT), ischaemia & necrotizing infections”

• Pseudomembranous colitis - plaque-like adhesion of yellow fibrinopurulent-necrotic debris & mucus (pseudomembrane) to damaged colonic mucosa
• Ischaemic colitis – ? mucosal membranes
• Crohn disease – no membranes
• UC – no membranes
• Infectious colitis
o Campylobacter – mucosal neutrophil infiltrate; crypt neutrophil infiltrate & crypt abscesses
o Shigella – haemorrhagic & ulcerated mucosa, pseudomembranes may be present
o Yersinia – likes ileocaecal region; aphthous ulcers & granulomas may form in the mucosa (like in Crohns)

185
Q

202.Acute colitis with membranes besides pseudomembranous?
1.Ischaemic colitis
2.Ulcerative colitis
3.Shigella
4.Crohn disease

A

a. Ischaemic colitis T listed in Robbins under PMC, but not described in the ischaemic section. Given above, best answer.

  1. Acute colitis with membranes besides pseudomembranous?
    a. Ischaemic colitis T listed in Robbins under PMC, but not described in the ischaemic section. Given above, best answer.
    b. Ulcerative colitis F
    c. Shigella T haemorrhagic & ulcerated mucosa, pseudomembranes may be present (Robbins p800 8e). Pseudomembranes occasionally present (Gattuso).
    d. Crohn disease F
186
Q

203.Which is least likely to develop GIT cancer?
a. Cathartic colon
b. Acanthosis nigricans
c. Celiac disease
d. Pernicious anaemia
e. Partial gastrectomy

A

a. Cathartic colon F can’t find ↑ risk anywhere

  1. Which is least likely to develop GIT cancer?
    a. Cathartic colon F can’t find ↑ risk anywhere
    b. Acanthosis nigricans T reported in association with a number of cancers, particular gastrointestinal malignancies (eg, gastric, hepatocellular) and lung cancer. See Goljan p554.
    c. Celiac disease T ↑ risk of T-cell lymphoma, SB adenocarcinoma, oesophageal SCC, colorectal carcinoma, HCC
    d. Pernicious anaemia T ↑ risk gastric cancer
    e. Partial gastrectomy T ↑ risk gastric cancer
187
Q

204.Which is most correct for oesophageal diseases?
1. Scleroderma has reflux oesophagitis
2. Scleroderma shows squamous metaplasia
3. Scleroderma shows adenomatous metaplasia
4. Scleroderma shows columnar metaplasia
5. Ulceration in achalasia

A
  1. Scleroderma has reflux oesophagitis T most correct answer?
    *LW: agree
  2. Which is most correct for oesophageal diseases..
  3. Scleroderma has reflux oesophagitis T most correct answer?
  4. Scleroderma shows squamous metaplasia F
  5. Scleroderma shows adenomatous metaplasia F in Barrett’s get intestinal (columnar) metaplasia
  6. Scleroderma shows columnar metaplasia T when Barrett’s oesophagus occurs (intestinal metaplasia with Goblet cells)
  7. Ulceration in achalasia F

Progressive atrophy and collagenous fibrous replacement of the muscularis. Mucosal thinning, ulceration & scarring throughout GIT.
GIT disease in scleroderma usually most severe in the oesophagus (rubber hose lower oesophagus – patulous LOS with atonic lower 2/3) – GOR arises due to dysmotility leading to Barrett oesophagus, lower oesophageal stricture, aspiration pneumonitis & Candida.

Oesophagus (J Clin Gastroenterol. 2006 Oct;40(9):769-75.)
Progressive systemic sclerosis (PSS) causes smooth muscle atrophy and fibrosis of the distal two-thirds of the esophagus. Motility studies show reduced-amplitude or absent peristaltic contractions in this region and normal or decreased lower esophageal sphincter pressure. Complications include strictures found in 17% to 29% of patients and Barrett esophagus is 0% to 37%. Esophageal disease correlates with pulmonary involvement but not with disease in the stomach or intestines.

188
Q

205.Which is most correct
1.Barrett oesophagus is defined by squamous metaplasia
2.Barrett oesophagus is defined by columnar metaplasia
3.Barrett oesophagus is defined by adenomatous metaplasia
4.Scleroderma oesophagus is characterized by gastroesophageal reflux
5.Achalasia is characterized by erosive oesophagitis

A

*LW - Unsure regarding options 2 and 4:
4. Scleroderma oesophagus is characterized by gastroesophageal reflux: don’t know if its “characterized by” GOR, more a complication (common) secondary to SM atrophy of lower 2/3 and sphincter +/- fibrosis replacement.

  1. Barrett oesophagus is defined by columnar metaplasia: RObbins states defined by intestinal metaplasia (columnar epithelium) within the oesophageal squamous mucosa, for which Goblet cells define intestinal metaplasia and are thus needed for the Dx of Barrett (Robbins 8e)

**LJS - I would favour (2.) Barrett’s defined by columnar metaplasia. Intestinal metaplasia: presence of metaplastic columnar mucosa above GOJ (replaced squamous epithelium). While scleroderma can have reflux, I would say it is characterised by smooth muscle atrophy/fibrosis causing marked oesophageal dilatation. Reflux is one of the consequences of this

  1. Which is most correct
    a. Barrett oesophagus is defined by squamous metaplasia F Google search – talks of urinary tract neoplasia only
    b. Barrett oesophagus is defined by columnar metaplasia F? defined by intestinal metaplasia (columnar epithelium) within the oesophageal squamous mucosa – Goblet cells define intestinal metaplasia & are needed for the Dx of Barrett (Robbins 8e)
    c. Barrett oesophagus is defined by adenomatous metaplasia F
    d. Scleroderma oesophagus is characterized by gastroesophageal reflux ?T don’t know if its “characterized by” GOR??
    e. Achalasia is characterized by erosive oesophagitis F too tight

Path notes – Nov 2001
Barrett: “metaplasia from squamous to columnar-type usually in lower oesphagus”.
Scleroderma: “Often the initial manifestation is DILATATION of the oesophagus… weak or absent peristalisis… often lower oesophageal stricture from virulent oesophagitis”.

Robbins 8e
Barrett: “Barrett esophagus is a complication of chronic GERD that is characterized by intestinal metaplasia within the esophageal squamous mucosa. Diagnosis of Barrett esophagus requires both endoscopic evidence of abnormal mucosa above the gastroesophageal junction and histologically documented intestinal metaplasia. Goblet cells, which have distinct mucous vacuoles that stain pale blue by H&E and impart the shape of a wine goblet to the remaining cytoplasm, define intestinal metaplasia and are necessary for diagnosis of Barrett esophagus.”

UTD – 2012
Barrett’s esophagus is the condition in which any extent of metaplastic columnar epithelium that predisposes to cancer development replaces the stratified squamous epithelium that normally lines the distal esophagus. The condition develops as a consequence of chronic gastroesophageal reflux disease (GERD) and predisposes to the development of adenocarcinoma of the esophagus.

AJR – April 2001
In the esophagus, (scleroderma) may be manifested by severe esophageal dysmotility with absent primary peristalsis in the smooth muscle portion of the esophagus, a patulous gastroesophageal junction with massive gastroesophageal reflux, reflux esophagitis, peptic strictures, Barrett’s esophagus, and even esophageal adenocarcinomas

189
Q

206.Obstruction in diverticular disease due to
1. Muscularis propria hypertrophy
2. Diverticulitis
3. Adhesions
4. Vesicocolic fistula
5. Circular muscle hypertrophy

A

a. Muscularis propria hypertrophy ?T adjacent muscularis propria may be thickened, fibrotic & chronically inflamed (Gattuso, DDx in surgical pathology p370)

  1. Obstruction in diverticular disease due to
    a. Muscularis propria hypertrophy ?T adjacent muscularis propria may be thickened, fibrotic & chronically inflamed (Gattuso, DDx in surgical pathology p370)
    b. Diverticulitis ?T can cause partial obstruction and secondary strictures (SK). TW: colonic obstruction occurs in 12% of cases of diverticulitis. Partial obstruction may occur from a circumferential or focally thickened and inflamed bowel wall, or an inflamed diverticulum with pericolic inflam change. Chronic inflammation can result in stricture formation, leading to luminal narrowing
    c. Adhesions ?T can occur secondary to diverticulitis, and can then later cause obstruction
    d. Vesicocolic fistula F complication of diverticulitis (in 14% of cases), most common type of fistula (also colovaginal, coloenteric, colocutaneous).
    e. Circular muscle hypertrophy ?T exaggerated peristalsis often induces muscular hypertrophy in affected segments, with unusually prominent taenia coli and circular muscle bundles. Circular muscle is part of the muscularis propria!

All could be true, except D. Difficult to pick the best answer, maybe A? (previously answer given was E but option A is a new answer & maybe is more true)

Clin Colon Rectal Surg. 2004 August; 17(3): 147–153.
Striking changes are seen in both the circular muscle layer and taeniae in patients with diverticular disease.14 Early necropsy studies describe increased bowel wall thickness in patients with diverticular disease12 and although this was initially felt to be from muscular hypertrophy, multiple subsequent studies have shown that there is no evidence of either hypertrophy or hyperplasia. Whiteway and Morson studied the muscularis propria in patients with uncomplicated diverticulosis and found that the taenia was thickened secondary to elastin deposition.15 There was an increase of over 200% in elastin in these patients compared with controls. As the elastin was laid down in a contracted form, shortening of the taeniae led to a bunching of the circular muscle, leading to the accordion-like or so-called concertina-like effect on the bowel. Thus, the remarkable muscular thickening found in association with diverticular disease represented an exaggerated contraction of normal colonic myocytes.14 Furthermore, subsequent studies have shown that muscular thickening extends only two thirds of the way around the circumference of the bowel, between the mesenteric and antimesenteric taeniae.12,16 This anatomical isolation of the muscle spasm corresponds with the areas of heaviest concentration of diverticula, as described earlier.

WJI: I’d chose e. The statdx article repeatedly talks about this

190
Q

Severe small bowel ischaemia, least associated with:
1. MI
2. SMA thrombus
3. Bechets
4. PAN
5. SMA embolous

A

• A = MI = T = global hypoperfusion
• B = SMA thrombosis = T
• C = Behcets = F = Behçet’s disease is characterized by recurrent oral aphthae and any of several systemic manifestations including genital aphthae, ocular disease, skin lesions, gastrointestinal involvement, neurologic disease, vascular disease, or arthritis. Believed to be due to vasculitis. (UTD)
• D = PAN = T = medium-vessel necrotising vasculitis. Ischemia due to vasculitis affects the small intestine more commonly than other areas of the gastrointestinal tract. In addition, a subset of patients have predominantly mesenteric arterial involvement with little or no clinical evidence of extraintestinal vasculitis. (UTD)
• E = SMA embolus = T

191
Q

Most common complication of H pylori:
1. MALT
2. Erosive gastritits
3. DU
4. GU

A

ANS = C duodenal ulcer (if chronic gastritis was an option, this would be the answer)

Helicobacter gastritis is a primary infection of the stomach and is the most frequent cause of chronic gastritis. Helicobacter pylori gastritis typically begins as a diffuse antral gastritis, which subsequently spreads to the gastric corpus if untreated.  common cause of chronic gastritis, but not of erosive gastritis (more due to alcohol, NSAIDs, chemo).

In the absence of NSAID use, H. pylori is associated with gastric ulcers ∼60-65% of the time.
In the absence of NSAIDs, H. pylori is the cause of duodenal ulcers in ∼70-100% of cases.
Peptic ulcers are four times more common in the proximal duodenum than in the stomach.

192
Q

Thickened folds stomach - preserved structure:
1. lymphoma

A

Causes of thickened gastric folds
• H. pylori gastritis
• ZES
• Crohn disease (with ulcers)
• Lymphoma
• Menetrier disease
• Superficial spreading (flat) gastric carcinoma
• Varices (pseudo-fold thickening)

193
Q

1st line Tx for MALT of stomach
1. chemo
2. Antibiotics
3. resection

A

ANS = B = antibiotics

The most striking evidence linking H. pylori gastritis to MALToma is that eradication of the infection with antibiotics induces durable remissions with low rates of recurrence in most patients.

194
Q

211.Scleroderma which is not a feature?
A) Sclerodactyly.
B) Raynaud’s.
C) Dilatation of the proximal oesophagus with a distal stricture
D) Diverticula on the mesenteric side of small bowel
E) Barrett’s oesophagus in 30-40%.

A
  1. Scleroderma which is not a feature?
    A) Sclerodactyly. T
    B) Raynaud’s. T
    C) Dilatation of the proximal oesophagus with a distal stricture T/F dilated atonic distal 2/3 with patulous LOS; in later stages can get distal stricture from GOR
    D) Diverticula on the mesenteric side of small bowel T/F pseudodiverticuli/pseudosacculations on antimesenteric border (Primer/Mayo)
    E) Barrett’s oesophagus in 30-40%. ?T (stats?)

B&H: stiff dilated oesophagus that does not collapse with emptying, and wide gaping LES with free GOR. Despite free reflux, tight strictures of distal oesophagus are uncommon. Danhert says ‘fusiform stricture 4-5cm above GOJ from reflux oesophagitis.

Path notes (Adelaide): “Often the initial manifestation is DILATATION of the oesophagus… weak or absent peristalisis… often lower oesophageal stricture from virulent oesophagitis”.

Scleroderma – small bowel (Gore & Levine p304, Mayo ebook p226)
• Dilatation of duodenum & jejunum (Type I folds = thin, straight folds with dilated lumen)
• Prolonged SB transit time
• Hidebound bowel = bunching & crowding of SB folds with ↑ no. of folds (virtually pathognomonic of scleroderma)
• Pseudodiverticula/sacculations
o Gore/Levine & Dahnert = mesenteric side
o Mayo GI = antimesenteric
o StatDx & Eisenberg = not specified which side!
o Top 3 = antimesenteric

Oesophagus (J Clin Gastroenterol. 2006 Oct;40(9):769-75.)
Progressive systemic sclerosis (PSS) causes smooth muscle atrophy and fibrosis of the distal two-thirds of the esophagus. Motility studies show reduced-amplitude or absent peristaltic contractions in this region and normal or decreased lower esophageal sphincter pressure. Complications include strictures found in 17% to 29% of patients and Barrett esophagus is 0% to 37%. Esophageal disease correlates with pulmonary involvement but not with disease in the stomach or intestines.

195
Q

212.ZE syndrome- Which is false? (Repeat)
a. Hypervascular pancreatic mass.
b. Gastrinoma.
c. Peptic ulcer disease.
d. Lymphoma in the gastric folds
e. Atrophic Gastritis.

A

e. Atrophic Gastritis. - F - hypergastrinaemia causes gastric gland hyperplasia (mainly parietal cells in fundus). Zollinger Ellison syndrome is a cause of hypertrophic gastropathy, as is Menetrier disease.

  1. ZE syndrome- Which is false? (Repeat)
    a. Hypervascular pancreatic mass. - T - islet cell tumor
    b. Gastrinoma. - T - islet cell gastrin-secreting tumor of the pancreas
    c. Peptic ulcer disease. - T - multiple in 10%; solitary ulcer in 90%; recurrent or intractable; unusual locations (postbulbar)
    d. Lymphoma in the gastric folds - F - complications include: malignant islet cell tumor (in 60%), liver mets with continued gastric secretion, perforated DU/jejunal ulcer, oesophageal stricture with reflux, obstruction, GI bleeding, gastric carcinoids (esp in MEN1)
    e. Atrophic Gastritis. - F - hypergastrinaemia causes gastric gland hyperplasia (mainly parietal cells in fundus). Zollinger Ellison syndrome is a cause of hypertrophic gastropathy, as is Menetrier disease.
196
Q

213.Non-caseating granulomas (Repeat)
a. Ulcerative colitis
b. TB
c. Crohn disease
d. Pseudomembranous colitis
e. Ischaemic colitis

A

c. Crohn disease T non caseating granulomas (50-60%) composed of epithelioid cells, a rim of lymphocytes, and multinucleated giant cells

  1. Non-caseating granulomas (Repeat)
    a. Ulcerative colitis F no granulomas
    b. TB F caseating granulomas
    c. Crohn disease T non caseating granulomas (50-60%) composed of epithelioid cells, a rim of lymphocytes, and multinucleated giant cells
    d. Pseudomembranous colitis F Pseudomembrane plaque-like adhesions of fibrinopurulent necrotic debris and mucus to damaged mucosa
    e. Ischaemic colitis F
197
Q

214.Not associated with Ulcerative Colitis:
f. Various options

A
  1. Not associated with ulcerative colitis
  • Extracolonic manifestations of UC: iritis, erythema nodosum, primary sclerosing cholangitis (5%), cholangiocarcinoma, chronic active hepatitis, spondylitis, peripheral arthritis, steroid related (AVN)